JB Learning Practice Tests

¡Supera tus tareas y exámenes ahora con Quizwiz!

Which of the following assessment findings should alert the paramedic that a patient with a closed lower extremity fracture is developing compartment syndrome? A) The pain is greater than one would expect for the injury B) The extremity becomes increasingly warmer and pinker C) Distal pulses are bounding and reflexes are hyperactive D) The pain subsides during passive stretch of the extremity

Compartment syndrome occurs when excessive pressure builds up in between the muscle and fascia (osteofascial compartment) and inhibits blood flow. The syndrome can occur following a fracture, crush injury, or venomous snakebite. Signs and symptoms of compartment syndrome include pain, pallor, pulselessness, parasthesia, paralysis, and poikilothermia (cold extremity). The pain is often disproportionate to the injury and is exacerbated by passive stretch of the extremity. Prompt transport of the patient is essential. A fasciotomy is required in order to relieve pressure from the osteofascial compartment and restore circulation. This is a surgical procedure and is not performed in the prehospital setting.

Which of the following injury mechanisms and clinical findings would MOST likely warrant transport to a facility that provides the highest level of trauma care? A) Motorcycle crash; pelvic instability; systolic BP of 100 mm Hg B) Fall from a standing position; no loss of consciousness; Glasglow Coma Scale of 14 C) Rollerblade accident; humeral fracture; heart rate of 100 beats/min D) Small-caliber gunshot wound to the calf; heart rate of 110 beats/min

(A) According to the 2011 CDC guidelines for the field triage of injured patients, the following injury mechanisms or clinical findings warrant transport to highest level of care in a defined trauma system: Glasgow Coma Scale less than or equal to 13; systolic BP less than 90 mm Hg; respiratory rate less than 10 or greater than 29 breaths/min (or the need for ventilatory support); all penetrating injuries to the head, neck, torso, and extremities proximal to the elbow or knee; chest wall instability or deformity (eg, flail chest); two or more proximal long bone fractures; crushed, degloved, mangled, or pulseless extremity; amputation proximal to the wrist or ankle; pelvic fractures; open or depressed skull fractures; and paralysis. The patient who crashed his motorcycle is the only patient who meets the trauma triage criteria.

Which of the following victim or bystander reactions would MOST likely escalate the stress associated with a mass-casualty incident? A) Anxiety B) Depression C) Blind panic D) Conversion hysteria

(C) In a situation involving multiple casualties, such as a train derailment, building collapse, or natural disaster (ie, tornado, flood, earthquake), both victims and bystanders may react by becoming dazed, disorganized, or overwhelmed. The American Psychiatric Association has identified five categories of reactions in such circumstances: anxiety, blind panic, depression, overreaction, and conversion hysteria. In general, people with these reactions should be removed from the scene; they can increase the stress of an already stressful situation. Typical signs of anxiety include sweating, trembling, weakness, nausea, and sometimes vomiting. People experiencing anxiety can recover fully within a few minutes and provide useful assistance if properly directed. Depression is seen in the individual who sits or stands in a numbed, dazed state. The depressed bystander needs to be brought back to reality and removed from the scene. People who overreact tend to talk compulsively, joke inappropriately, become overly active, and race from one task to another without accomplishing anything useful. The person who is overreacting needs to be removed from the area where casualties are being treated. In conversion hysteria, the patient or bystander subconsciously converts anxiety into a bodily dysfunction; he or she may be unable to see or hear or may become paralyzed in an extremity. The most worrisome reaction is blind panic, in which the individual's judgment seems to disappear entirely. Blind panic is particularly dangerous because it is "catchy," and may cause mass panic among others present. For this reason, a panicky bystander must be separated quickly from others and, if at all possible, placed under the supervision of a calmer person.

Which of the following 12-lead ECG findings signifies a left bundle branch block? A) QRS duration of 126 ms; terminal S wave in lead aVL B) QRS duration of 124 ms; terminal S wave in lead V1 C) QRS duration of 122 ms; terminal S wave in lead V6 D) QRS duration of 128 ms; terminal R wave in lead V1

A QRS duration of greater than 120 ms (0.12 seconds [3 small boxes]) in the context of a supraventricular rhythm indicates a bundle branch block. A left bundle branch block (LBBB) is characterized by a QRS duration of greater than 120 ms, a terminal S wave in lead V1 (the last wave of the QRS complex is an S wave), and a monophasic R wave in lead I. A right bundle branch block (RBBB) is characterized by a QRS duration of greater than 120 ms, a terminal R wave in lead V1 (the last wave of the QRS complex is an R wave), and slurred S waves in lead I.

After administering epinephrine to a woman in anaphylactic shock, her condition improves. The paramedic should recognize that her clinical improvement is because epinephrine: A) constricts the blood vessels and dilates the bronchioles. B) decreases the heart rate, which improves cardiac output. C) blocks histamine receptors and stops the allergic reaction. D) causes fluid to shift back into the cell, which reduces edema.

A. The life-threatening effects of anaphylactic shock are vasodilation (causes hypotension) and bronchoconstriction (impairs breathing). Epinephrine stimulates alpha and beta receptors; as such, it constricts the blood vessels (alpha-1 stimulation) and dilates the bronchioles (beta-2 stimulation). Through beta-1 stimulation, epinephrine increases the heart rate. The therapeutic effects of epinephrine during anaphylactic shock are vasoconstriction (increases blood pressure) and bronchodilation (improves breathing). Diphenhydramine (Benadryl) is given after epinephrine in anaphylactic shock; it blocks histamine receptors and stops the allergic reaction.

You are evaluating a regular cardiac rhythm in lead II. The rate is 90 beats/min, the QRS complexes consistently measure 0.16 seconds, and inverted P waves are seen immediately following each QRS complex. The rhythm described is MOST characteristic of a/an: A) wandering atrial pacemaker with a bundle branch block. B) accelerated junctional rhythm with ventricular aberrancy. C) ectopic atrial rhythm with a ventricular conduction delay. D) second-degree AV block with abnormal ventricular conduction.

B. A junctional rhythm is characterized by inverted P waves in lead II. If seen, the inverted P waves precede or follow the QRS complex. At a rate of 90 beats/min, the rhythm is further defined as an accelerated junctional rhythm. QRS complexes greater than 0.12 seconds (120 ms) indicate aberrant (abnormal) ventricular conduction (ie, bundle branch block). A wandering atrial pacemaker is characterized by P waves that precede each QRS complex, but vary in morphology. An ectopic atrial rhythm is also characterized by P waves of varying morphologies as well as varying PR intervals. A second- or third-degree AV block should be suspected when there are more P waves than QRS complexes.

You are assessing an adult patient with a head injury. He opens his eyes only when you speak to him, and when you ask him his name, he replies slowly by saying "No thank you." When you apply a painful stimulus by pinching his arm, he pulls his arm away from you. What is this patient's Glasgow Coma scale score? A) 9 B) 10 C) 11 D) 12

The Glasgow Coma Scale (GCS) is a valuable tool when performing neurologic assessments of a head-injured patient. A single GCS assessment cannot reliably capture the patient's clinical progression; therefore, you should obtain a baseline GCS score and frequently (at least every 5 minutes) reassess it. Furthermore, when relaying the patient's GCS to the emergency department physician, state where the patient's deficits are; don't simply give a number. Your patient's present GCS score is a 10. He opens his eyes only when you speak to him; under the "eye opening" component of the GCS, this equates to a score of 3. Using inappropriate words - stating "No thank you" when you ask him his name - equates to a score of 3 under the "verbal response" component of the GCS. Withdrawal from pain (ie, pulling his arm away when a painful stimulus is applied) equates to a score of 4 under the "motor response" component of the GCS. A GCS score of 13 to 15 indicates a mild traumatic brain injury (TBI), a score of 8 to 12 indicates a moderate TBI, and a score of 3 to 8 indicates a severe TBI.

Which of the following statements regarding Cheyne-Stokes respirations is correct? A) Cheyne-Stokes respirations always indicate increased intracranial pressure and brainstem dysfunction. B) It is common to observe Cheyne-Stokes respirations in patients who have overdosed on narcotic drugs. C) They are not considered ominous unless grossly exaggerated or in the context of a traumatic brain injury. D) They are characterized by an irregular pattern, rate, and depth of respiration with intermittent apneic periods.

Cheyne-Stokes respirations are more of a high-brain function. Many deep sleepers or intoxicated people exhibit this respiratory pattern. The depth of breathing (or volume of snoring) gradually increases, then decreases (crescendo-decrescendo), followed by an apneic period. Despite their abnormal appearance, Cheyne-Stokes respirations are not considered ominous unless they are grossly exaggerated or occur in the context of a traumatic brain injury. Biot's respirations, also called ataxic respirations, are characterized by an irregular pattern, rate, and depth of breathing with intermittent apneic periods; they indicate a severe brain injury. Overdose of a narcotic (opiate) drug, such as heroin or morphine, would be expected to cause slow, shallow breathing secondary to depression of the respiratory centers in the brain.

Proprioception is the ability to sense the position, location, orientation and movement of a part of the body in relation to another. Muscles, tendons, joints and the inner ear contain proprioceptors, which relay positional information to the brain. The brain then analyzes this information and provides us with a sense of body orientation and movement. The inability of a patient to comprehend questions is called receptive aphasia. A spinal cord injury results in compromised sensory and motor functions distal to the injury site, not proximal. The condition in which the body's temperature assumes that of the environment is called poikilothermia.

A. According to current emergency cardiac care (ECC) guidelines, cardiac arrest in a patient with moderate hypothermia (core body temperature [CBT] between 86° F [30° C] and 93° F [34° C]) should be treated with CPR, a single defibrillation every 2 minutes for V-Fib or pulseless V-Tach, and cardiac medication administration as indicated. However, the medications should be spaced at longer than standard intervals. In moderate hypothermia, the patient's basal metabolic rate (BMR) is slow, thus the onset and duration of action of medications (ie, vasopressors, antidysrhythmics) will be longer. Hyperventilation should be avoided in any patient. Not only does it hyperinflate the lungs and impede venous return to the heart, it facilitates heat loss, not production. If the cardiac arrest patient is severely hypothermic (CBT < 86° F [30° C]), cardiac medications should be withheld, and defibrillation, if indicated, should be attempted one time only.

Which of the following ECG findings indicates a pathologic delay at the AV node? A) P-R interval of 0.28 seconds B) QRS complex of 0.16 seconds C) P-R interval less than 0.12 seconds D) P waves of varying morphologies

A. Normally, there is a physiologic delay of an impulse at the AV node that allows the atria to empty into the ventricles. On the ECG, this manifests as a P-R interval - the period of time that includes atrial depolarization and the delay at the AV node - that is between 0.12 and 0.20 seconds (120 to 200 ms). A pathologic delay at the AV node, such as what occurs with a first-degree AV block, would manifest with a P-R interval that is greater than 0.12 seconds (120 ms) in duration. By contrast, A P-R interval that is less than 0.12 seconds indicates that an impulse is traversing the AV node too fast or is bypassing it altogether, such as what occurs with Wolff-Parkinson-White (WPW) syndrome, a preexcitation syndrome in which the electrical impulse follows accessory pathways around the AV node (bundle of Kent) and prematurely depolarizes the ventricles. A wide (> 0.12 seconds [120 ms]) QRS complex indicates an intraventricular conduction delay, such as a bundle branch block. P waves that vary in morphology (appearance) indicate more than one atrial pacemaker site; an example of this is an ectopic atrial rhythm.

A patient experiencing a right ventricular infarction would be expected to present with: A) hypertension and tachycardia B) ST elevation in leads II, III, and aVF. C) greater than 2-mm ST depression in lead V1. D) severe pulmonary edema and hemoptysis.

B. A right ventricular infarction (RVI) should be suspected when a patient presents with ECG changes indicative of an inferior wall injury pattern (equal to or greater than 1-mm ST elevation in leads II, III, and aVF; reciprocal ST depression and T wave inversion in leads I and aVL) AND has equal to or greater than 1-mm ST elevation in lead V4R when a right-sided 12-lead ECG is obtained. Patients experiencing an RVI are preload dependent and often present with hypotension; therefore, vasodilators (eg, nitroglycerin, morphine) should be avoided. Instead, IV fluid boluses should be given to maintain adequate perfusion. Other signs of an RVI include jugular venous distention and peripheral edema. Pulmonary edema and coughing up blood (hemoptysis) are indicative of left ventricular failure.

Which of the following statements regarding fetal circulation in utero is correct? A) The ductus venosus returns deoxygenated blood from the fetus to the mother via the placenta. B) The ductus arteriosus and foramen ovale divert blood flow away from the fetus's lungs. C) The foramen ovale shunts oxygenated blood from the left atrium directly into the right atrium. D) Oxygenated blood is delivered from the mother to the fetus via the two umbilical arteries.

B. In utero, the fetus's lungs are full of fluid; therefore, its blood must be oxygenated by the mother via the umbilical cord and placenta. The foramen ovale shunts oxygenated blood from the fetus's right atrium directly into the left atrium, thus diverting blood flow away from the lungs. The ductus arteriosus connects the pulmonary artery and the aorta; it also diverts blood flow away from the lungs. Following delivery, the foramen ovale and ductus arteriosus constrict, which, in combination with an increase in fetal pulmonary circulation and clearance of fluid from the fetus's alveoli, allows the newborn to oxygenate its own blood. The ductus venosus transports oxygenated blood from the placenta to the fetus; it empties directly into the fetus's vena cava. The two umbilical arteries return deoxygenated blood and other wastes from the fetus to the placenta, where it is subsequently eliminated by the maternal circulation.

A 59-year-old woman with a history of Grave's disease presents with an altered mental status. Her skin is hot to the touch and her pulse rate is 160 beats/min. These findings are MOST consistent with: A) myxedema. B) thyrotoxic crisis. C) addisonian crisis. D) Cushing syndrome.

B. Thyrotoxic crisis (thyroid storm, thyrotoxicosis) is a hypermetabolic clinical syndrome caused by critically high levels of the thyroid hormones T3 (triiodothyronine) and T4 (thyroxine). Signs and symptoms include high fever (as high as 105° F to 106° F [40.5° C to 41.1° C]), hypertension, and profound tachycardia. Untreated, it can lead to cardiac arrest. Thyrotoxic crisis may occur in conjunction with Grave's disease, the most severe and common cause of hyperthyroidism, or it may occur if a patient takes too much of their prescribed thyroid supplement (ie, levothyroxine). Advanced hypothyroidism (myxedema) is a hypometabolic clinical syndrome caused by a deficiency of T3 and T4; the patient's signs and symptoms are not consistent with myxedema. Addisonian crisis, an acute manifestation of adrenal insufficiency, typically occurs after the abrupt cessation of corticosteroid therapy (ie, prednisone, hydrocortisone); this would not explain the patient's hypermetabolic state. Cushing syndrome, caused by excessive cortisol production by the adrenal glands, can also cause a hypermetabolic state. However, given the patient's history of Grave's disease, this is less likely.

You are called to an assisted living facility for a sick resident. The patient, a 70-year-old woman, reports tinnitus and difficulty concentrating. The patient's neighbor, who is present at the scene, tells you that the patient has consumed five or six cups of ice over the past hour. You should suspect: A) acute leukemia. B) chronic anemia. C) polycythemia. D) lymphoma.

B. Tinnitus (ringing in the ears), a craving for ice, and difficulty concentrating are hallmark findings of chronic anemia. Anemia is defined as a deficiency of red blood cells or hemoglobin. Other findings may include a headache, dizziness, tachycardia, pallor, and shortness of breath. Polycythemia, an overproduction of red blood cells, causes flushing of the skin; it is not commonly associated with the clinical signs that this patient is exhibiting. Leukemia and lymphoma are types of cancer that commonly manifest with persistent weakness, fever, and swollen lymph nodes (lymphadenopathy).

A 44-year-old male was struck across the face with a steel pipe during an assault. Your assessment reveals marked swelling and ecchymosis to his midfacial area. He has nasal discharge, blurred vision, and is unable to follow your finger above the midline with his left eye. These clinical findings are MOST indicative of a/an: A) Le Fort III fracture. B) orbital skull fracture. C) basilar skull fracture. D) cribriform plate fracture.

B. Your patient has an orbital fracture, also called a blowout fracture. The orbits are cone-shaped fossae that enclose and protect the eyes. In addition to the eyeball (globe) and muscles that move it, the orbit contains blood vessels, nerves, and fat. A direct blow to the eye may fracture the orbital floor because the bone is thin and breaks easily. An orbital fracture results in transmission of forces away from the eyeball itself to the bone. Blood and fat then leak into the maxillary sinus. The patient may complain of visual disturbances, have massive nasal discharge, and may lose sensation above the eyebrow or over the cheek secondary to associated nerve damage. Periorbital ecchymosis (raccoon eyes) is often present. Fractures of the inferior orbit are the most common and can cause paralysis of upward gaze; the patient's injured eye is unable to follow your finger above the midline. A Le Fort III fracture (craniofacial disjunction) involves a fracture of all the midfacial bones, separating the entire midface from the cranium. Basilar skull fractures are associated with high-energy trauma, but usually occur following diffuse impact to the head. Signs of a basilar skull fracture include cerebrospinal fluid (CSF) drainage from the ear (CSF otorrhea) and bruising over the mastoid bone behind the ear (Battle's sign). Fractures of the cribriform plate of the ethmoid bone, a bone that separates the nasal cavity from the brain, are often caused by a sharp upward blow to the nose; bony fragments are forced through the cribriform plate into the meninges or brain. CSF drainage from the nose (CSF rhinorrhea) is common in patients with a cribriform plate fracture.

A 60-year-old woman, who has been taking high doses of prednisone for several months to treat her rheumatoid arthritis, presents with weakness and fatigue that has progressively worsened. On appearance, her face appears swollen. You should be MOST suspicious that this patient has: A) Grave's disease. B) Addison's disease. C) Cushing's syndrome. D) Adrenal insufficiency.

C. Cushing's syndrome is caused by excess cortisol production by the adrenal glands or by excessive use of cortisol or similar steroid (glucocorticoid) hormones. Tumors of the pituitary gland or adrenal cortex can stimulate the production of cortisol, for example, and lead to Cushing's syndrome. Administration of large amounts of corticosteroid hormones (ie, prednisone, methylprednisolone, dexamethasone, hydrocortisone) to treat conditions such as rheumatoid arthritis, asthma, and systemic lupus can also cause Cushing's syndrome. Regardless of the cause, excess cortisol causes characteristic changes in many body systems. Protein synthesis is impaired so that body proteins are broken down, which leads to loss of muscle fibers with resultant muscle weakness. Common signs and symptoms of Cushing's disease include weakness and fatigue, depression and mood swings, darkening of the skin (acanthosis) on the neck, and weight gain - especially on the abdomen, face ("moon face"), neck, and upper back. Primary adrenal insufficiency - also known as Addison's disease - is caused by atrophy or destruction of the adrenal glands, leading to a deficiency of all the steroid hormones produced by these glands. Grave's disease, a disease process associated with hyperthyroidism, occurs when the thyroid gland produces excess thyroid hormones.

Disequilibrium syndrome manifests with signs and symptoms of: A) severe hypokalemia. B) high serum potassium levels. C) increased intracranial pressure. D) excessive catecholamine release.

C. During dialysis, the concentration of urea in the blood is lowered rapidly, while the solute concentration of the cerebrospinal fluid (CSF) remains high. Water, of course, moves by osmosis from a solution of lower concentration into a solution of higher concentration. Thus, as a consequence of dialysis, water initially shifts from the bloodstream into the CSF, thereby increasing intracranial pressure (ICP). In such a circumstance, the patient may experience typical signs and symptoms of mildly increased ICP, including nausea, vomiting, headache, and confusion. After a few hours, the fluid will re-equilibrate between the bloodstream and CSF, and the patient's symptoms will spontaneously resolve. In the field, however, it may be impossible to distinguish between disequilibrium syndrome and subdural hematoma, to which dialysis patients are particularly prone. Signs and symptoms of hyperkalemia (high serum potassium levels) are commonly encountered in patients with chronic renal failure, especially if the patient's potassium intake is increased or if a dialysis treatment is missed. Hyperkalemic patients commonly present with profound muscular weakness; on the ECG, peaked T waves and prolonged QRS complexes are commonly observed. Hypokalemia (low serum potassium level) may occur as a consequence of overaggressive dialysis, and would most likely be encountered during or immediately following a dialysis cycle. Hypokalemic patients are often hypotensive and commonly present with cardiac dysrhythmias (usually bradycardias). Tachycardia and hypertension - signs of excessive catecholamine release - are not commonly associated with dialysis.

You are ventilating a severely dehydrated apneic 70-year-old male with a history of end-stage emphysema. In order to minimize the risk of lowering his cardiac output and blood pressure, you should: A) use a manually triggered ventilation device. B) hyperventilate the patient at 20 to 24 breaths/min. C) adjust the ventilation rate to allow complete exhalation. D) use a device that provides positive-end expiratory pressure.

C. In patients with severe COPD (ie, end-stage emphysema) and increased resistance to exhalation, you should attempt to prevent air trapping as this may cause inadvertent generation of intrinsic positive end-expiratory pressure (also called "auto-PEEP"). In hypovolemic patients - as with your severely dehydrated patient - auto-PEEP may significantly reduce cardiac output and blood pressure. Adjusting the ventilation rate to approximately 6 to 8 breaths/min - which will allow for complete exhalation - can prevent this. Manually-triggered ventilation devices (eg, demand valve) should not be used in any patient with pulmonary air trapping; use of such devices may result in widespread alveolar rupture and/or a pneumothorax.

A 5-year-old male presents with labored breathing, stridor, and high fever. The child's mother tells you that his symptoms rapidly progressed over the last few hours. You should suspect: A) croup. B) asthma. C) epiglottitis. D) bronchiolitis.

C. The child's clinical presentation is consistent with epiglottitis, a bacterial infection that causes severe swelling of the epiglottis. Signs and symptoms of epiglottitis include a rapid progression of respiratory distress, high fever, stridor (indicates upper airway swelling), painful swallowing, and drooling. Croup is caused by a viral infection and typically has a slower onset of symptoms; a hallmark of croup is a cough that sounds like a barking seal. Most children with croup have a low-grade fever. Asthma, a lower airway obstruction, is not the likely cause of the child's symptoms. Asthma presents with wheezing, not stridor. Bronchiolitis is a lower airway infection that most commonly affects infants; it is associated with wheezing and low-grade fever.

Following a spinal injury, your patient experiences a loss of proprioception. This means that: A) the patient is unable to comprehend simple questions. B) the body's temperature assumes that of the environment. C) motor function is decreased proximal to the site of the injury. D) the patient is unaware of one body part in relation to another.

D. Proprioception is the ability to sense the position, location, orientation and movement of a part of the body in relation to another. Muscles, tendons, joints and the inner ear contain proprioceptors, which relay positional information to the brain. The brain then analyzes this information and provides us with a sense of body orientation and movement. The inability of a patient to comprehend questions is called receptive aphasia. A spinal cord injury results in compromised sensory and motor functions distal to the injury site, not proximal. The condition in which the body's temperature assumes that of the environment is called poikilothermia.

The clinical presentation of thyroid storm MOST closely resembles that of: A) myxedema. B) ketoacidosis. C) heroin overdose. D) amphetamine use.

D. Thyroid storm is a life-threatening condition that may occur in patients with hyperthyroidism. Hyperthyroidism is a condition in which the thyroid gland produces too much of the thyroid hormones triiodothyronine (T3) and thyroxine (T4). T3 and T4 play a key role in controlling the body's basal metabolic rate (BMR). When produced in excess, these thyroid hormones produce signs and symptoms of adrenergic hyperactivity (ie, tachycardia, sweating, palpitations, anxiety). In thyroid storm, these clinical signs would be more severe, including profound tachycardia, dysrhythmias, agitation and paranoia, and cardiovascular collapse. Amphetamines also cause adrenergic hyperactivity; of the choices listed, amphetamine use and thyroid storm would produce the most similar clinical signs. Heroin, a narcotic, causes central nervous system depression, resulting in a decreased level of consciousness, hypoventilation, bradycardia, and hypotension. Advanced hypothyroidism, called myxedema, results from a severe deficiency of T3 and T4. In contrast to thyroid storm, myxedema does not present with signs of adrenergic hyperactivity.

Prehospital treatment for a patient in ventricular fibrillation who has a core body temperature of less than 86° F (30° C) includes: A) doubling the dose of all medications. B) limiting defibrillation to one attempt only. C) administering lidocaine instead of amiodarone. D) hyperventilation with warm humidified oxygen.

(B) Although severely hypothermic patients in cardiac arrest usually does not respond to conventional ACLS therapies, one shock (360 monophasic joules or equivalent biphasic) can be attempted if the patient is in V-Fib or pulseless V-Tach. If the patient does not respond to one shock, further defibrillation attempts should be deferred; the paramedic should focus on providing high-quality CPR with minimal interruptions, airway management (do NOT hyperventilate the patient), rewarming per local protocol, and prompt transport. If the patient's core body temperature is less than 86° F (30° C), cardiac medications (ie, epinephrine, amiodarone, lidocaine) will be ineffective because the patient's metabolic rate is so low. Furthermore, they can accumulate to toxic levels if given repeatedly. For these reasons, cardiac medications should be withheld until the patient is properly rewarmed at the hospital.

A 64-year-old man presents with an acute onset of left-sided weakness, right-sided facial droop, and slurred speech. He is conscious, but confused. You should be MOST suspicious for: A) right-sided hemorrhagic stroke. B) right-sided ischemic stroke. C) left-sided ischemic stroke. D) left-sided hemorrhagic stroke.

(B)Recalling that the right side of the brain controls the left side of the body and vice versa, this patient's clinical presentation is most consistent with an ischemic stroke to the right cerebral hemisphere. Ischemic strokes, caused by a blocked cerebral artery, typically present with confusion, unilateral weakness (hemiparesis) or paralysis (hemiplegia) on the opposite (contralateral) side of the stroke, a facial droop on the same (ipsilateral) side as the stroke, and dysarthria (slurred or poorly articulated speech). Pupillary abnormalities, if observed, typically occur on the ipsilateral side because of optic nerve crossover in the brain. The patient's clinical presentation is less consistent with a hemorrhagic stroke, which typically presents with a sudden, severe headache; a rapid loss of consciousness; and signs of increased intracranial pressure (ie, hypertension, bradycardia, breathing abnormalities). A hemorrhagic stroke is the result of a ruptured cerebral artery (ie, an aneurysm).

If a small child accidentally pulled a pot of boiling water off of the stovetop, you would MOST likely encounter: A) partial-thickness burns with a multidirectional splash pattern to the anterior aspect of the body. B) redness and blistering to the posterior aspect of the body with a bidirectional splash pattern. C) a downward splash pattern of superficial and partial-thickness burns to the anterior aspect of the body. D) superficial and partial-thickness burns isolated to the anterior trunk without evidence of a splash pattern)

(C) When assessing an injured child, the paramedic must carefully assess the injury pattern and compare that to the mechanism described by the parent or caregiver. A knowledge of typical (and atypical) injury patterns, along with other information obtained at the scene (ie, length of time between the injury and the call to 9-1-1, inconsistencies among caregivers regarding the details of the event), will enable the paramedic to recognize the signs of potential abuse. If a small child accidentally pulled a pot of boiling water from the stovetop, you would expect to see a downward splash pattern, mainly to the anterior aspect of the body because the child moves away from the pain of the boiling water. Boiling water typically causes superficial and partial-thickness burns; full-thickness burns are uncommon. A multidirectional splash pattern to the anterior or posterior aspect of the body is more consistent with hot water being thrown at the child, and should make you suspicious that the burn was intentionally inflicted. Document this finding factually and objectively, and report your suspicions to the emergency department physician.

When passing an endotracheal tube in between the vocal cords, the paramedic should recall that: A) the trachea makes an acute left angle just beyond the vocal cords. B) the left mainstem bronchus is shorter and straighter than the right. C) the right mainstem bronchus takes a more acute angle than the left. D) the trachea descends into the chest cavity just beyond the vocal cords.

(D)The trachea is not straight; it descends into the chest cavity just beyond the vocal cords. As soon as the tip of the ET tube passes between the vocal cords, the paramedic should rotate the tube to the right; this will direct the tip of the tube downward and allow it to descend down the trachea. If the tip of the ET tube is directed upward, it will hit the anterior wall of the trachea and will not pass. If the ET tube is inserted too far, it will come to rest in the right mainstem bronchus, which is shorter and takes a less acute angle than the left; this would be unhealthy for the patient if unrecognized.

A person's level of consciousness is regulated by the: A) diencephalon. B) limbic system. C) medulla oblongata. D) reticular activating system.

A person's level of consciousness is regulated by the reticular activating system (RAS), which is at the level of the midbrain - a part of the brain stem. The thalamic region of the diencephalon acts as a relay center; it filters important signals from routine signals. The hypothalamic region of the diencephalon regulates functions such as emotions (pleasure), body temperature, and interaction with the endocrine system. The limbic system is the region of the brain where the emotions of rage and anger are generated. The medulla oblongata is part of the brain stem. It regulates functions such as heart rate, respiratory rate, and blood pressure. The medulla and the pons (portion of the brain stem that regulates respiratory patterning and depth) function as the respiratory centers of the brain.

At the end of ventricular relaxation, the left ventricle contains 110 mL of blood. This is referred to as the: A) Preload B) Afterload C) Stroke volume D) Cardiac output

A. The volume of blood in the ventricle at the end of relaxation (diastole) is called the preload, or end-diastolic volume. Afterload refers to the resistance that the ventricle must contract against. Stroke volume is the amount of blood ejected from the ventricle per contraction. Cardiac output is the volume of blood ejected from the ventricle each minute; it is a product of the stroke volume and heart rate.

Which of the following is an absolute contraindication for fibrinolytic therapy? A) Subdural hematoma 3 years ago B) BP of 170/100 mm Hg on presentation C) Current use of anticoagulant medication D) Ischemic stroke within the last 12 months

A. According to current emergency cardiac care (ECC) guidelines, absolute contraindications for fibrinolytic therapy include ANY prior intracranial hemorrhage (ie, subdural, epidural, intracerebral hematoma); known structural cerebrovascular lesion (ie, arteriovenous malformation); known malignant intracranial tumor (primary or metastatic); ischemic stroke within the past 3 months, EXCEPT for acute ischemic stroke within the past 3 hours; suspected aortic dissection; active bleeding or bleeding disorders (except menses); and significant closed head trauma or facial trauma within the past 3 months. Relative contraindications (eg, the physician may deem fibrinolytic therapy appropriate under certain circumstances) include, a history of chronic, severe, poorly-controlled hypertension; severe uncontrolled hypertension on presentation (SBP > 180 mm Hg or DBP > 110 mm Hg); ischemic stroke greater than 3 months ago; dementia; traumatic or prolonged (> 10 minutes) CPR or major surgery within the past 3 weeks; recent (within 2 to 4 weeks) internal bleeding; noncompressible vascular punctures; pregnancy; prior exposure (> 5 days ago) or prior allergic reaction to streptokinase or anistreplase; active peptic ulcer; and current use of anticoagulants (ie, Coumadin).

You are assessing a 59-year-old woman who complains of chest pressure. When you are looking at her list of medications, you note that she takes Vasotec. What type of medication is this? A) ACE inhibitor B) Beta-blocker C) Calcium channel blocker D) Parasympathetic blocker

A. Enalapril maleate (Vasotec) is an ACE (angiotensin converting enzyme) inhibitor that is used to treat hypertension. Angiotensin II, a potent chemical produced by the kidneys that causes vasoconstriction, is formed from angiotensin I in the blood by the angiotensin converting enzyme. ACE inhibitors block the activity of this enzyme, which decreases the production of angiotensin II. As a result, the blood vessels dilate and blood pressure is reduced. Beta blockers, which are also used to treat hypertension, include drugs such as metoprolol (Lopressor), propranolol (Inderal), and atenolol (Tenormin), among others. Calcium channel blockers are also used to treat hypertension, and include drugs such as diltiazem (Cardizem), verapamil (Calan; Isoptin), and amlodipine (Norvasc), among others. Atropine sulfate is a parasympathetic blocker (vagolytic) that is used to treat patients with hemodynamically unstable bradycardia.

You are called to a residence for a 39-year-old woman, who, according to her husband, is "not acting right." She is confused, is experiencing hallucinations, and is repetitively smacking her lips. Which of the following should you suspect? A) Complex partial seizure B) Simple partial seizure C) Focal motor seizure D) Generalized seizure

A. Seizures are classified as being generalized or partial. Your patient's presentation is consistent with a complex partial seizure. Partial seizures affect a limited part of the brain and are further divided into simple partial and complex partial. Simple partial seizures involve movement (frontal lobe) or sensations (parietal lobe) to one part of the body. A focal motor seizure is a simple partial seizure with localized motor activity. There may be spasm or clonus (jerking) of one muscle or muscle group, which may remain localized or may spread to adjacent muscles (Jacksonian march). Complex partial seizures involve changes in level of consciousness. The patient can become confused, lose alertness, experience hallucinations, or may be unable to speak. Automatisms, such as lip smacking, chewing, swallowing, may occur with complex partial seizures. Generalized seizures affect the entire brain. Tonic/clonic seizures (full body jerking movements), absence seizures (freezing or staring), and pseudoseizures (tonic/clonic, but caused by a psychiatric mechanism) are examples of generalized seizures.

A 29-year-old female presents with lower abdominal pain and minor vaginal bleeding. She recently had a dilation and curettage (D&C) procedure following a spontaneous abortion. Her blood pressure is 108/68 mm Hg, pulse is 110 beats/min, and respirations are 20 breaths/min. Her temperature is 102.4° F. You should be MOST suspicious for: A) endometritis. B) an ovarian cyst. C) endometriosis. D) pelvic inflammatory disease.

A. Endometritis is an inflammation of the inner uterine lining (endometrium), usually due to infection. It may also occur following gynecologic surgery (ie, D&C), abortion, or intrauterine device use. Symptoms include malaise, fever, constipation, vaginal bleeding or discharge, and lower abdominal or pelvic pain. Endometritis may lead to septic shock if untreated. Endometriosis is a condition in which endometrial tissue grows outside the uterus, usually on the abdominal and pelvic organs. Symptoms include pain localized in the lower back, pelvis, and abdomen; painful bowel movements during menses, fatigue, escalating menstrual cramps, and heavy vaginal bleeding. Fever is usually absent. Pelvic inflammatory disease (PID) is an infection of the female reproductive organs; it occurs almost exclusively in sexually active women. In PID, disease-causing organisms enter the vagina during intercourse and invade the uterine cavity. The infection may spread to the fallopian tubes and ovaries. PID presents with severe abdominal pain and fever. An ovarian cyst is a fluid-filled sac that forms on or within an ovary. In a functional cyst, the ovaries form tiny sacs (cysts) during the menstrual cycle to hold the eggs. Once the egg matures, it is released from the sac, which subsequently dissolves. If the sac seals itself after release of the egg, fluid accumulates in it, the cyst begins to grow (corpus luteum cyst), and the patient experiences abdominal pain on the side of the cyst; fever is usually absent.

You are assessing the cardiac rhythm of a 9-year-old child who says his chest is "fluttering." P waves are not visible, the QRS complexes measure 70 ms in duration, the heart rate is 200 beats/min, and there is no variation in the R-R intervals when the child moves. These findings are MOST consistent with: A) reentry supraventricular tachycardia. B) ventricular tachycardia. C) ectopic atrial tachycardia. D) sinus tachycardia.

A. Most narrow-complex tachycardias in infants and children are sinus tachycardia. However, some children present with reentry supraventricular tachycardia (SVT). In children older than 1 year of age, reentry SVT is characterized by a ventricular rate greater than 180 beats/min (> 220 beats/min in infants), narrow (less than or equal 90 ms) QRS complexes, and no variability in the R-R intervals with movement or activity. Furthermore, children with reentry SVT typically do not present with a history that is compatible with common causes of sinus tachycardia (ie, fever, hypovolemia); in some cases, the child may have a history of reentry SVT. In sinus tachycardia, the heart rate in children is usually less than 180 beats/min (< 220 beats/min in infants), the R-R intervals vary with activity or movement, and the child has a history that is compatible with common causes of sinus tachycardia (ie, fever, hypovolemia). Ventricular tachycardia (V-Tach) is uncommon in children, but should be suspected if the QRS complexes measure greater than 90 ms in duration. Ectopic atrial tachycardia (EAT), also uncommon in children, is characterized by P waves of varying morphologies. Diagnosing EAT in children with tachycardia is often not possible, however, because the P waves are usually not visible.

When arterial oxygen levels in the body fall, chemoreceptors in the brain send messages to the diaphragm via the: A) phrenic nerve. B) vagus nerve. C) brainstem. D) medulla.

A. When low oxygen levels are detected by the chemoreceptors in the blood, messages are sent to the diaphragm via the phrenic nerve, which originates in between the third and fifth cervical vertebrae. Injury to the cervical spine in this area can sever the phrenic nerve and result in respiratory paralysis. The chemoreceptors in the brain are located within the medulla oblongata, a part of the brainstem.

A 39-year-old female, who is 35 weeks pregnant, presents with acute abdominal pain, irregular contractions, and vaginal bleeding. She is confused, her blood pressure is 70/48 mm Hg, pulse is 120 beats/min and weak, and respirations are 24 breaths/min and shallow. Her abdomen is tender to palpation and her uterus feels rigid. She is MOST likely experiencing: A) uterine rupture. B) placenta previa. C) abruptio placenta. D) spontaneous abortion.

Abruptio placenta is the partial or complete detachment of the placenta beyond the twentieth week of gestation. Risk factors include multiparity, maternal trauma, maternal cocaine use, and maternal hypertension. It commonly presents with acute onset vaginal bleeding and abdominal pain in the third trimester. Bleeding may be disproportionate to the degree of shock because the placenta often falls over the cervical opening after detachment, containing much of the blood in the uterus. Other signs include abdominal pain and uterine rigidity. Placenta previa is the partial or complete implantation of the placenta over the cervical opening. Placenta previa typically presents with painless vaginal bleeding, although pain may be present in some cases. Uterine rupture typically presents with acute, tearing abdominal pain, active labor, and shock; abdominal palpation often reveals rigidity and pain, and in some cases, fetal parts. Previous cesarean sections or abdominal trauma are risk factors for uterine rupture. A spontaneous abortion is the spontaneous termination of pregnancy before the 20th week of pregnancy; this patient is 35 weeks pregnant.

Treatment for a patient who has a pulse and a documented core body temperature of 92.8° F includes: A) passive rewarming only. B) prophylactic antidysrhythmic therapy. C) 20 mL/kg boluses of warm normal saline. D) passive and active external rewarming.

According to current emergency cardiac care (ECC) guidelines, a patient with mild hypothermia (CBT of 93.2° F to 96.8° F [34° C to 36° C]) should be treated with passive rewarming, which involves removing wet clothing, applying warm blankets, and allowing the patient's CBT to rise naturally. If the patient has moderate hypothermia (CBT of 86° F to 93.2° F [30° C to 34° C]), passive and active external rewarming should be performed. Active external rewarming involves the use of heating blankets, or radiant heat from hot packs placed in the groin, neck, and axillae. Active internal rewarming, including the administration of warm (109.4° F [43° C]) IV fluids, warm-water peritoneal lavage, and esophageal rewarming tubes, should be performed if the patient has severe hypothermia (CBT of less than 86° F [30° C]). In most cases, active internal (core) rewarming will take place at the hospital. Prophylactic antidysrhythmic therapy is not indicated for hypothermic patients.

Which of the following ECG lead configurations is correct? A) To assess lead II, place the negative lead on the right arm and the positive lead on the left leg. B) To assess lead I, place the positive lead on the right arm and the negative lead on the left arm. C) To assess lead III, place the negative lead on the left leg and the positive lead on the right arm. D) To assess lead III, place the negative lead on the right arm and the positive lead on the left leg.

According to the Einthoven triangle, lead I is assessed by placing the negative (white) lead on the right arm and the positive (red) lead on the left arm. Lead II is assessed by placing the negative lead on the right arm and the positive lead on the left leg. Lead III is assessed by placing the negative lead on the left arm and the positive lead on the left leg.

Which of the following is a defining factor in the transition from human immunodeficiency virus (HIV) infection to acquired immunodeficiency syndrome (AIDS)? A) Antibodies are detected in the blood B) Fever that lasts greater than 7 days C) Transient increase in the T-cell count D) Development of opportunistic infections

Acquired immunodeficiency syndrome (AIDS) is the end-stage disease process caused by infection with the human immunodeficiency virus (HIV). Although it can obviously be stated that all patients with AIDS are HIV positive, it cannot be stated that all HIV-positive patients have AIDS. A diagnosis of AIDS is made when the number of T-helper lymphocytes (CD4 cells) fall below a certain level, or when the patient develops one of a specific group of opportunistic infections (also called AIDS-defining or AIDS-related conditions). Such opportunistic infections include Pneumocystis carinii pneumonia, cytomegalovirus, red or purple malignant skin lesions called Kaposi sarcoma, atypical tuberculosis, and cryptococcol meningitis. The point at which antibodies to the virus are detected in the blood is called seroconversion; this usually occurs within the first 3 months following exposure to HIV. With antiretroviral therapy, patients can remain HIV-positive for many years without showing any evidence of infection.

Which of the following clinical presentations is MOST consistent with an acute ischemic stroke involving the left cerebral hemisphere? A) Dysarthria, confusion, left side hemiparesis, left side facial droop B) Dysphasia, confusion, right side hemiparesis, left side facial droop C) Decerebrate posturing, asymmetric pupils, hypertension, bradycardia D) Aphasia, lethargy, right side hemiparesis, right side facial droop

Acute ischemic strokes represent approximately 75% of all strokes. Each cerebral hemisphere controls functions on the contralateral (opposite) side of the body; therefore, sensory and motor deficits (ie, hemiparesis, paralysis, facial droop) are observed on the side of the body opposite the stroke. Other common signs of acute ischemic stroke include dysarthria (difficulty in articulating words, resulting in slurred speech), dysphasia (difficulty speaking or understanding), aphasia (inability to speak or understand), and mental status changes. In contrast to acute ischemic stroke, acute hemorrhagic stroke (caused by a ruptured cerebral artery) typically presents with more ominous signs that tend to progress rapidly, which include a sudden, severe headache that is followed by a rapid decline in level of consciousness. Because bleeding is occurring within the brain, intracranial pressure increases, resulting in signs such as decorticate (flexor) or decerebrate (extensor) posturing, asymmetric or bilaterally dilated pupils, and Cushing's triad (hypertension, bradycardia, abnormal respiratory pattern).

Which of the following signs is LEAST suggestive of an underlying arterial injury in a patient with a closed extremity fracture? A) Disproportionate pain B) Weak or absent distal pulse C) Pulsatile expanding hematoma D) A palpable thrill over the injury

An arterial injury may occur in conjunction with an open or closed fracture. In patients with a closed fracture, signs of an underlying arterial injury include a diminished or absent distal pulse, a pulsatile expanding hematoma (indicates rapid internal bleeding from an artery), and a palpable thrill (vibration) over the injury site that correlates with the patient's pulse. Searing or burning pain that is out of proportion to the injury is a common early sign of compartment syndrome - a condition associated with crush injuries, fractures, or dislocations - in which pressure builds within the osteofascial compartment (the space between groups of muscles surrounded by fascia) of an extremity.

After being struck in the head with a baseball bat, a 9-year-old boy immediately loses consciousness. On your arrival, he is conscious but confused. Shortly into your assessment, he becomes unresponsive. Which of the following injuries has this child MOST likely sustained? A) Epidural hemorrhage B) Subdural hemorrhage C) Cerebral contusion D) Severe concussion

An epidural hemorrhage, which is usually the result of damage to the middle meningeal artery, produces a loss of consciousness immediately after impact, after which the patient typically has a brief return of consciousness. As the arterial bleeding begins to increase the pressure within the cranium, however, the patient's mental status rapidly deteriorates. Subdural hemorrhages are usually venous in nature; signs and symptoms may not present for up to 24 hours or longer.

Which of the following clinical presentations is MOST consistent with dissection of the ascending aorta? A) Tearing abdominal pain unrelieved by analgesia, pulse deficit in the femoral arteries, lightheadedness, blood in the stool B) Sudden onset of lower back pain that radiates to the groin, urge to defecate, pain is constant and moderate in severity C) Gradual onset of chest pressure that increases in severity over time, hypotension, tachycardia, bilaterally weak radial pulses D) Acute tearing pain in between the scapulae, blood pressure discrepancy between arms, maximal pain severity from the onset

Aortic dissection occurs when the layers of the aorta undergo destructive changes, resulting in an aneurysm (weakening and ballooning of the arterial wall). In dissection of the ascending aorta, the patient typically experiences an acute onset of ripping, tearing, or stabbing pain in the anterior chest or in between the scapulae. In some patients, it may be difficult to differentiate the pain of acute aortic dissection from that of acute myocardial infarction (AMI); however, a number of distinctive features may help. The pain of an AMI is often preceded by prodromal symptoms (eg, nausea, weakness, sweating). Although pain from an AMI is acute, it gradually intensifies over time and is typically described as a squeezing or pressure sensation. By contrast, the pain of aortic dissection is acute, is of maximal intensity from the onset, and is usually described as a ripping, tearing, or stabbing feeling. Other signs and symptoms depend on the extent and location of the dissection. In dissections of the ascending aorta, one or more of the vessels of the aortic arch may be compromised. Disruption of blood flow through the innominate artery, for example, is likely to produce a difference in blood pressure between the arms. The onset and pain characteristics of abdominal aortic dissection are similar to those of ascending aortic dissection; however, the pain typically begins in the abdomen or lower back. Pulse deficits in the femoral arteries may be present, and if the aneurysm is leaking blood into the retroperitoneal space, the patient may complain of an urge to defecate and exhibit signs of shock.

Which of the following is an abnormal physiologic process that occurs at the capillary level during shock? A) Precapillary sphincter relaxation in response to lactic acid buildup B) Minimal capillary blood flow, which results in aerobic metabolism C) Sustained pre- and postcapillary constriction in response to hypoxia D) Postcapillary sphincter relaxation that causes capillary fluid depletion

As perfusion decreases, cellular ischemia occurs. Minimal blood flow passes through the capillaries, causing cellular conversion from aerobic metabolism to anaerobic metabolism, which produces lactic acid and can quickly lead to metabolic acidosis. With less circulation, blood stagnates in the capillaries. The precapillary sphincter relaxes in response to the buildup of lactic acid, vasomotor center failure, and increased carbon dioxide levels. The postcapillary sphincters remain constricted, causing the capillaries to become engorged with fluid. The capillary sphincters - circular muscular walls that constrict and dilate - regulate blood flow through the capillary beds and are under the control of the autonomic nervous system. Among other factors (ie, heat, cold), capillary sphincters respond to an increased demand for oxygen and the need for waste removal. Thus, regulation of blood flow is determined by cellular need and is accomplished by vascular constriction or dilation, working in tandem with capillary sphincter constriction or dilation.

A 4-year-old boy has a high fever and deep, rapid respirations. The child's mother states that she thinks her child got into the medicine cabinet. Which of the following medications has the child MOST likely ingested? A) Acetaminophen B) Aspirin C) Codeine D) Ibuprofen

Aspirin (acetylsalicylic acid [ASA]) toxicity produces a high fever (hyperpyrexia) and metabolic acidosis. As the respiratory system attempts to rid the body of the excess acid and hydrogen ions, the patient's respirations become deep and rapid. Acetaminophen (APAP) toxicity result in liver damage or failure. Codeine is a narcotic and will depress the central nervous system, causing hypoventilation, bradycardia, and hypotension. Ibuprofen toxicity typically result in gastrointestinal damage.

Following ingestion of a toxic dose of acetaminophen, right upper quadrant abdominal pain typically begins within: A) 12 to 24 hours B) 24 to 72 hours C) 72 to 96 hours D) 4 to 14 days

B. Acetaminophen, the active ingredient in Tylenol, can cause liver failure and death if a toxic dose is ingested. A unique aspect of acetaminophen toxicity is that its signs and symptoms appear in four distinct stages. Stage I (less than 24 hours) symptoms include nausea, vomiting, anorexia, pallor, and malaise. Stage II (24 to 72 hours) symptoms include right upper quadrant (RUQ) abdominal pain and abdominal tenderness to palpation. Stage III (72 to 96 hours) symptoms include metabolic acidosis, renal failure, coagulopathies, and recurring GI symptoms. During Stage IV (4 to 14 days [or longer]), recovery slowly begins or liver failure progresses and the patient dies. The antidote for acetaminophen toxicity is acetylcysteine (Acetadote); ideally, it should be given less than 8 hours following ingestion.

A patient struck the steering wheel when his vehicle crashed into a tree. Assessment reveals respiratory distress, diminished breath sounds to the left lower hemithorax, a scaphoid abdomen, and jugular venous distention. This clinical presentation is MOST consistent with a: A) massive hemothorax. B) ruptured diaphragm. C) pericardial tamponade. D) tension pneumothorax.

B. Injuries to the diaphragm, although rare, result from both blunt and penetrating trauma. Blunt force trauma following a high-speed motor vehicle crash is one of the most common mechanisms of injury. During a frontal impact, the unrestrained patient strikes the steering wheel or column, causing a sudden increase in intraabdominal pressure that may tear the diaphragm. Because the left side of the diaphragm is embryonically weak and the right side is protected by the liver, ruptures - particularly those caused by blunt trauma - are more common on the left side. In most cases, a portion of the large intestine enters the thoracic cavity through the diaphragmatic tear, causing ventilatory impairment by compressing the lung. In larger diaphragmatic tears, cardiac output may be decreased secondary to impaired ventricular filling, resulting in hypotension, tachycardia, and increased jugular venous pressure. Signs of diaphragmatic rupture include labored breathing, asymmetric chest wall movement, jugular venous distention, and a scaphoid abdomen (the abdominal wall is sunken and presents with a concave rather than a convex contour). In some cases, bowel sounds can be heard in the lower to middle part of one of the hemithoraces; however, unilaterally diminished breath sounds are more common.

A construction worker fell approximately 30 feet and landed on a concrete surface. He is responsive to pain only; has rapid, shallow respirations; and has a slow, weak pulse. As your partner maintains manual stabilization of his head and assists his ventilations, you perform a rapid head-to-toe assessment. The patient has a closed deformity to his right femur, numerous abrasions, an open deformity to his right humerus, and deformity in the area of the fifth thoracic vertebra. His blood pressure is 74/50 mm Hg. What is the MOST likely pathophysiology of this patient's clinical presentation? A) Internal hemorrhage due to multiple long bone fractures B) Relative hypovolemia due to impaired adrenergic function C) Shock due to failure of the parasympathetic nervous system D) Vascular failure secondary to increased alpha-1 stimulation

B. Neurogenic shock, often the result of a spinal injury, is caused by impaired sympathetic nervous system tone, which results in vasodilation. In neurogenic shock, vascular smooth muscle does not receive impulses that cause it to contract. As a result, vessels distal to the spinal injury dilate, increasing the size of the vascular space. The normal volume blood can no longer fill the enlarged vascular space. Perfusion of organs and tissues becomes inadequate and hypotension occurs, even though blood loss has not (relative hypovolemia). Bradycardia occurs due to unopposed parasympathetic stimulation and an absence of catecholamine release. Pallor and diaphoresis, signs of sympathetic stimulation and shunting of blood from the periphery, is often observed above the level of the spinal injury. However, the skin below the level of the injury is warm and dry due to vasodilation caused by a loss of vasomotor control. Relative to other types of shock, neurogenic shock presents differently; the patient is hypotensive, but does not have the classic signs of pallor, diaphoresis, and tachycardia.

Which of the following findings would lead the paramedic to suspect pericarditis when assessing a 40-year-old male with chest pain and no cardiac history? A) The patient's age and absence of a cardiac history B) The pain decreases when the patient sits forward C) Widespread ST depression of greater than 2 mm D) Greater than 3-mm ST elevation in leads V2 and V3

B. Pericarditis is an inflammation of the pericardium, the lining that surrounds the heart. It can have many causes, including infection, injury, chronic diseases, and radiation treatment. The pain associated with pericarditis often begins acutely and commonly radiates to the back, in between the scapulae. ECG findings include diffuse ST elevation, an absence of reciprocal changes, and intact precordial R wave progression. Unlike the pain associated with acute myocardial infarction, the pain associated with pericarditis is typically sharp in nature, is made worse when the patient takes a deep breath and/or lies flat, and subsides when the patient sits up. A patient's age and absence of a cardiac history does not make pericarditis more or less likely. Widespread ST depression suggests diffuse myocardial ischemia, not pericarditis. ST elevation that is isolated to an individual lead group (for example, leads V2 and V3) indicates acute myocardial injury.

A deep partial-thickness burn is characterized by: A) pain, erythema, and destruction of the superficial epidermal layer. B) edema, blister formation, and decreased sensation around the burn. C) charred or leathery skin, absence of pain, and destruction of the dermis. D) blister formation, intense pain, and injury that extends through the dermis.

B. Superficial (first-degree) burns are characterized by pain and erythema (redness), and damage that is limited to the superficial layer of the epidermis. Partial-thickness (second-degree) burns are categorized as being superficial partial-thickness and deep partial-thickness burns. In both types of partial-thickness burn, injury extends through the epidermis and into the dermis and fluid infiltrates in between the dermis and epidermis, creating edema and blisters. Unlike a superficial partial-thickness burn, however, a deep partial-thickness burn damages the basal layer of the dermis. As a result, sensation in and around the burn is decreased owing to damage to the nerve endings in the basal layer. A full-thickness (third-degree) burn damages the entire epidermis and dermis, including the nerve endings; this explains why full-thickness burns are usually painless. A full-thickness burn is characterized by charred, white, or leathery skin.

When assessing the pupils of a patient with a severe closed head injury, you note that they are bilaterally dilated and nonreactive. What does this specifically indicate? A) Bleeding above the dura mater B) Pressure on the oculomotor nerve C) Severe injury to the hypothalamus D) Herniation of the upper brainstem

B. The oculomotor nerve (cranial nerve III) arises from the midbrain and exits the brain to each eye. It controls the upper eyelid muscle, which raises the eyelid; the extraocular muscle, which moves the eye inward; and the pupillary muscle, which constricts the pupil. Damage to or pressure on this nerve will cause the pupils to dilate and fail to constrict when light is shone into them. Common causes of this include increased intracranial pressure, stroke (both ischemic and hemorrhagic), and cerebral hypoxia.

Which of the following assessment findings helps distinguish envenomation due to a black widow spider bite from an acute abdominal condition? A) Dizziness, nausea, vomiting, and diaphoresis B) Abdominal rigidity without palpable tenderness C) Hematemesis, dysphagia, and significant hypotension D) Fever, palpable abdominal tenderness, and tachycardia

B. The venom of a black widow spider contains a neurotoxin that causes severe muscles spasms and intense pain - especially of the abdomen. Unlike an acute abdominal condition, however, the patient's abdomen is typically not tender to palpation. Dizziness, nausea, vomiting, and diaphoresis are common in patients with an acute abdominal condition and black widow spider envenomation. Fever and tachycardia in conjunction with abdominal tenderness is consistent with acute abdominal conditions such as peritonitis and appendicitis, not envenomation from a black widow spider. Hematemesis, dysphagia, and hypotension are common assessment findings in patients with ruptured esophageal varices.

Which of the following cardiac rhythms is associated with bradycardia, and is characterized by regular R-R intervals and a greater ratio of P waves to QRS complexes? A) First-degree AV block B) Third-degree AV block C) Second-degree AV block type I D) Second-degree AV block type II

B. Third-degree AV block is caused by a complete block at the AV node. The SA node initiates impulses as usual; however, when they reach the AV node, they are blocked. Resultantly, the ventricles receive no electrical stimulus from the atria, so they initiate their own impulses, although at a much slower rate. On the ECG, this manifests as a bradycardic rhythm with more P waves than QRS complexes. The P-P intervals are regular (some P waves may not be visible because they are buried in a QRS complex), as are the R-R intervals; however, no relationship exists between a given P wave and QRS complex. Second-degree AV block type I (Wenckebach) is caused by a progressive delay at the AV node until an impulse is blocked from entering the ventricles. On the ECG, this manifests as a progressively lengthening P-R interval until a P wave is blocked (not followed by a QRS complex). At this point, the R-R interval becomes irregular, and the presence of this lone P wave increases the ratio of P waves to QRS complexes. Second-degree AV block type I may or may not be associated with bradycardia. Second-degree AV block type II is caused by an intermittent block at the AV node; it occurs when atrial impulses are not conducted to the ventricles. Unlike a second-degree AV block type I, however, a type II block is characterized by consistent P-R intervals of the P waves that are conducted. First-degree AV block is an abnormal delay at the AV node; on the ECG, this manifests with PR intervals greater than 0.20 seconds (120 ms) in duration. In first-degree AV block, all of the atrial impulses are conducted through the AV node and into the ventricles.

A 23-year-old man was working near a wood pile when he experienced a sudden, sharp pain in his leg. Your assessment reveals that his level of consciousness is decreased and he is experiencing intense abdominal spasms. This clinical presentation is MOST consistent with the bite of a: A) brown recluse spider. B) black widow spider. C) rattlesnake or other pit viper. D) coral snake.

B. This is a classic case of a black widow spider bite. The black widow spider typically can be found near wood piles or wood sheds. The patient will usually experience immediate sharp pain at the time of the bite, and then within a short period of time painful muscle spasms will develop in all of the major muscle groups, especially the abdomen. The black widow spider carries a neurotoxin in its venom, which explains the muscle spasms. If left untreated, central nervous system depression will continue and the patient will experience cardiovascular and respiratory system collapse. In contrast to a black widow spider bite, the bite of a brown recluse spider is usually painless, and the patient does not even realize he or she has been bitten until a red area with a small blister in the center of it appears several hours to a day later. Unlike the black widow spider, the brown recluse spider carries a cytotoxin (necrotoxin) in its venom. Cytotoxins cause tissue and cellular necrosis.

The baroreceptors in the aortic arch and carotid sinuses are extremely sensitive to: A) increases in arterial carbon dioxide. B) changes in arterial perfusion pressure. C) increases or decreases in the heart rate. D) fluctuations in the level of arterial oxygen.

Baroreceptors, also known as "pressure" receptors, are located within the carotid arteries and aorta. They are extremely sensitive to changes in arterial perfusion pressure (ie, blood pressure). When the baroreceptors sense a drop in arterial blood pressure, they send signals via the sympathetic nervous system, resulting in the release of catecholamines (epinephrine and norepinephrine). Catecholamines cause vasoconstriction, resulting in an increase in systemic vascular resistance, as well as increases in heart rate (positive chronotropy) and myocardial contraction force (positive inotropy). Central and peripheral chemoreceptors constantly monitor the pressure of gases (oxygen and carbon dioxide) in the blood, as well as the pH of the cerebrospinal fluid, and help regulate ventilation rate and depth.

You are caring for a 44-year-old female who experienced a closed head injury. She is responsive only to pain, has a respiratory rate of 8 breaths/min and irregular, a pulse of 120 beats/min and weak, and a blood pressure of 80/50 mm Hg. Which of the following statements regarding this patient is MOST correct? A) You should intubate the patient and give IV fluid boluses to maintain a systolic blood pressure of at least 90 mm Hg. B) You should suspect that the patient is experiencing neurogenic shock in addition to increased intracranial pressure. C) The patient's brain is herniating, so you should hyperventilate her with a bag-mask device at a rate of 20 breaths/min. D) Instead of giving IV fluid boluses, administer a vasopressor drug until her systolic blood pressure is at least 90 mm Hg.

Because of this patient's markedly decreased level of consciousness following her head injury, her airway should be definitively secured to prevent aspiration if she vomits. Because she is not completely unresponsive, sedation and neuromuscular blockade will likely be required to facilitate intubation. Once she has been intubated, ventilate her at a rate of 10 breaths/min. There is no evidence of brain herniation (eg, unresponsiveness, decerebrate posturing, asymmetric pupils), so do NOT hyperventilate her! Her vital signs - hypotension and tachycardia - are not consistent with an isolated closed head injury; they are more consistent with shock, probably from an occult hemorrhage. If an adult with a seemingly isolated head injury presents with signs of shock, look for other injuries! A single episode of hypotension (< 90 mm Hg) in the head-injured adult can cause cerebral ischemia and increase mortality significantly. Give isotonic crystalloid fluid boluses as needed to maintain a systolic BP of at least 90 mm Hg. It is unlikely that the patient is in neurogenic shock, which is characterized by hypotension and a slow (or relatively slow) heart rate; she is tachycardic, which indicates that her sympathetic nervous system is intact and is releasing catecholamines. Vasopressor drugs may be useful in treating patients with neurogenic shock that is refractory to IV fluid boluses; however, this patient is not in neurogenic shock.

Which of the following findings is observed with right bundle branch block? A) Monophasic R wave in lead 1 B) Underlying ventricular rhythm C) Terminal R wave in lead V1 D) QRS duration of less than 120 ms

C. Before a bundle branch block can be diagnosed, the underlying rhythm must be of a supraventricular origin and the QRS duration must be greater than 120 ms. Right bundle branch block (RBBB) is identified by a terminal R wave in lead V1 and slurred S waves in lead I. Left bundle branch block (LBBB) is identified by a terminal S wave in lead V1 and a monophasic R wave in lead I.

Which of the following vehicle impacts will create the greatest amount of kinetic energy? A) 160-lb patient who is traveling at 45 mph B) 170-lb patient who is traveling at 55 mph C) 150-lb patient who is traveling at 60 mph D) 180-lb patient who is traveling at 50 mph

C. Kinetic energy (KE) is a function of an object's mass (weight) and velocity (speed). The greater than amount of KE, the greater the potential for serious injury or death. The relationship between weight and speed as it affects KE is as follows: KE equals one-half the mass times the velocity square (KE = 1/2mv2). According to this formula, doubling the mass doubles the amount of KE; however, doubling the velocity quadruples the amount of KE. A 150-lb patient who strikes a fixed object while traveling at 60 mph will create 270,000 units of KE, as follows: 150 ÷ 2 (75) × 60 squared (3,600) = 270,000. A 160-lb patient traveling at 45 mph will create 162,000 units of KE, a 170-lb patient traveling at 55 mph will create 257,125 units of KE, and a 180-lb patient traveling at 50 mph will create 225,000 units of KE.

Easy bruising, lymph node enlargement, and splenomegaly are clinical manifestations of: A) anemia. B) lymphoma. C) leukemia. D) polycythemia.

C. Leukemia is cancer of the blood, and is caused by an abnormal proliferation (production by multiplication) of leukocytes (white blood cells) in the bone marrow. Leukemic cells impair the normal production of red blood cells (RBCs), white blood cells (WBCs), and platelets (thrombocytes); this results in anemia, leukopenia (low WBC count), and easy bleeding due to thrombocytopenia (low platelet count). In leukemia, excessive white blood cells accumulate in major organs (ie, spleen, liver, brain, and lymph), causing them to become enlarged (ie, splenomegaly [enlarged spleen], adenopathy [enlarged lymph nodes], hepatomegaly [enlarged liver]). Other signs and symptoms of leukemia include bone pain (due to increased pressure in the medullary canal of the bone), fever, fatigue, night sweats, and weight loss.

You would MOST likely encounter bradypnea in a patient who: A) ingested salicylates. B) is acutely hypoxemic. C) has metabolic alkalosis. D) took 5 mg of diazepam.

C. Metabolic alkalosis (pH > 7.45) often results in periods of bradypnea (abnormally slow respirations). This occurs due to compensation by the respiratory buffer system, which is attempting to retain carbon dioxide and hydrogen ions in order to lower the pH. Five milligrams of diazepam (Valium) is a therapeutic dose; central nervous system depression (ie, bradypnea, bradycardia, hypotension) would be unlikely at this dose, unless the patient co-ingested another central nervous system depressant (eg, opiates). When the body's chemoreceptors sense increased arterial CO2 levels (hypercarbia) or decreased O2 levels (hypoxemia), the respiratory centers in the brain send more messages to the respiratory muscles; as a result, respirations increase (tachypnea) in order to bring in more oxygen and eliminate more carbon dioxide. A patient who ingested salicylates (ie, acetylsalicylic acid [aspirin, ASA]) would present with tachypnea because the respiratory buffer system is attempting to eliminate excess hydrogen ions by increasing the rate and depth of breathing.

While treating a patient with severe pain, the paramedic accidentally gives the patient 10 mg of morphine when he meant to give 5 mg. As a result, the patient becomes bradycardic and hypotensive. The paramedic's action constitutes: A) nonfeasance. B) malfeasance. C) misfeasance. D) gross negligence.

C. Negligence is commonly divided into three categories: malfeasance, misfeasance, and nonfeasance. Malfeasance occurs when a paramedic performs an act that he or she was never authorized to perform, such as a medical intervention that is outside his or her scope of practice. Misfeasance occurs when a paramedic performs an act that he or she is legally permitted to do, but does so in an improper manner. For example, a paramedic administers a drug that is clearly within his or her scope of practice, but inadvertently calculates or administers the wrong dose. Nonfeasance occurs when the paramedic fails to perform an act that he or she is required or expected to perform. Failure to administer oxygen to a hypoxemic patient is an example of nonfeasance. Gross negligence is established if the paramedic's actions or inactions were found to be willful or wanton (malicious) under the law. In this case, the paramedic's action was accidental, not intentional. Therefore, gross negligence cannot be established.

Which of the following clinical presentations is consistent with significant insecticide exposure? A) Extreme hyperactivity and pupillary dilation B) Acute urinary retention and abdominal pain C) Excessive salivation and severe bradycardia D) A dry cough, tachycardia, and hypertension

C. Organophosphates, such as what is found in pesticides/insecticides and chemical nerve agents (VX, sarin, tabun, soman), deactivate acetylcholinesterase (AChE), an enzyme that regulates the degradation of acetylcholine (ACh). ACh is the chemical neurotransmitter of the parasympathetic nervous system. Without AChE, there is nothing to regulate ACh degradation; this would cause cardiovascular collapse secondary to massive parasympathetic stimulation. The clinical presentation of severe organophosphate toxicity can be recalled using the mnemonic DUMBELS, which stands for defecation, urination, miosis (pupillary constriction), bronchorrhea and bradycardia, emesis, lacrimation, and salivation. Treatment includes atropine sulfate, which blocks the effects of ACh, and pralidoxime chloride (2-PAM, Protopam), which reactivates AChE. A commercial auto-injector (DuoDote) is available; it contains both pralidoxime and atropine.

An older man is suddenly awakened in the middle of the night, gasping for air. He is extremely restless and pale, and is coughing up blood. His clinical presentation is MOST consistent with: A) unstable angina. B) gastrointestinal bleed. C) left side heart failure. D) right side heart failure.

C. Waking up in the middle of the night with severe difficulty breathing (paroxysmal nocturnal dyspnea [PND]) and coughing up blood or blood-tinged sputum (hemoptysis) are consistent with left-sided heart failure and pulmonary edema. Right-sided heart failure typically does not present with respiratory distress; it commonly manifests with jugular venous distention and peripheral edema. Shortness of breath and hemoptysis are not consistent with a gastrointestinal (GI) bleed; signs of a GI bleed include abdominal pain, vomiting up blood (hematemesis), which may be bright red or have a coffee-ground appearance; dark, tarry stools (melena); or bright red blood in the stool (hematochezia). Because left-sided heart failure can be caused by other factors, such as a long history of poorly-controlled hypertension, angina may or may not be present.

During your assessment of a patient with a suspected neurologic disorder, you ask him to shrug his shoulders and turn his head from side to side. Which of the following cranial nerves are you assessing? A) Trigeminal B) Vestibulocochlear C) Spinal accessory D) Glossopharyngeal

C. time and patient condition permits, assessment of the 12 pairs of cranial nerves should be performed. The spinal accessory nerve (XI), a motor nerve, controls shoulder and neck movements. Asking the patient to shrug his shoulders and turn his head from side to side assesses cranial nerve XI. The trigeminal nerve (V) provides motor control to the muscles of chewing and sensory control to the face, sinuses, and teeth. Asking the patient to clench his teeth and then lightly stroking your finger over his forehead and cheeks and asking him to identify where he is being touched assesses cranial nerve V. The vestibulocochlear nerve (VIII), a sensory nerve, controls hearing and balance perception. Checking a patient's hearing and asking him to stand on one leg (if safe to do so) assesses cranial nerve VIII. The glossopharyngeal nerve (IX) provides motor control to the throat and swallowing mechanism and sensory control to the tongue, throat and ear. Asking the patient to swallow assesses cranial nerve IX. Refer to your paramedic textbook regarding assessment of all of the cranial nerves.

An infant or child with cardiogenic shock: A) classically has a bradycardic rhythm on the cardiac monitor. B) often needs a significant volume of IV fluid to improve perfusion. C) requires an epinephrine infusion to increase myocardial contractility. D) often presents with increased work of breathing and an enlarged liver.

Cardiogenic shock (pump failure) is uncommon in children, but may be the result of congenital heart disease, myocarditis, or a dysrhythmia. Children in cardiogenic shock are listless or lethargic; are pale or mottled; and have cool, clammy skin. In addition, they also show signs of increased work of breathing owing to congestive heart failure and pulmonary edema. Impaired ventricular function causes an increase in central venous pressure; this manifests as an enlarged liver (hepatomegaly) and jugular venous distention (JVD). However, JVD is difficult to assess in infants; it is more appreciable in older children. In children with certain congenital heart diseases, their Sp02 may remain low despite high-flow oxygen; parents will often alert you of this. Unless you are certain of the diagnosis of cardiogenic shock (the child has congenital heart disease, is afebrile, and has no history of volume loss), err on the side of fluid resuscitation. Administer a single fluid bolus slowly, and monitor carefully to assess its effect. Increased work of breathing, a drop in Sp02, or worsening perfusion after a fluid bolus confirms your suspicion of cardiogenic shock. If your transport time is long, medical control may order a dopamine infusion to increase cardiac contractility and improve perfusion. Epinephrine has more pronounced chronotropic effects than dopamine, which may significantly increase cardiac oxygen consumption and demand; thus, it is not the drug of choice for patients with cardiogenic shock.

Which of the following parameters yields the lowest cerebral perfusion pressure? A) BP, 96/50 mm Hg, intracranial pressure, 5 mm Hg B) BP, 100/54 mm Hg; intracranial pressure, 10 mm Hg C) BP, 105/60 mm Hg; intracranial pressure, 20 mm Hg D) BP, 130/90 mm Hg; intracranial pressure, 25 mm Hg

Cerebral perfusion pressure (CPP) is the amount of pressure that it takes to push blood through the cerebral circulation and maintain blood flow and oxygen and glucose delivery to the cells of the brain. If the CPP drops below 60 mm Hg, cerebral perfusion will be compromised. Although the paramedic cannot measure CPP in the field, he or she should know how to calculate and interpret it. This knowledge will facilitate an appreciation of the disastrous effects that hypotension and/or increased intracranial pressure (ICP) can have on CPP. CPP decreases when the mean arterial pressure (MAP) decreases and/or the ICP increases (ICP should not exceed 15 mm Hg). Even a single episode of hypotension in the patient with a traumatic brain injury can be catastrophic. CPP is calculated as follows: CPP = MAP - ICP. MAP is calculated as follows: MAP = systolic BP + (diastolic BP × 2) ÷ 3. A BP of 105/60 mm Hg (MAP of 75) with an ICP of 20 mm Hg yields a CPP of 55 mm Hg, which is the lowest of the options listed.

You are assessing an injured football player who you suspect has injured his Achilles tendon. To determine if the Achilles tendon is intact, you should: A) let the injured leg hang dependent and observe for the presence of a foot drop. B) have the patient dorsiflex his foot and ask him if he feels pain in his calf muscles. C) stroke the sole of the foot with a blunt object and observe for upward toe movement. D) squeeze the calf muscles of the injured leg and observe for plantar flexion of the foot.

D. Rupture of the Achilles tendon usually occurs in athletes older than 30 years of age who are involved in start-and-stop sports such as basketball or football. The most immediate indications of Achilles tendon rupture are pain from the heel to the calf and a sudden inability for plantar flexion of the foot - extension of the ankle resulting in the forefoot moving away from the body. The Thompson test can be performed in the field to identify an Achilles tendon rupture. To perform this test, have the patient assume a prone position and then squeeze the calf muscles of the injured leg. If the foot plantar flexes while squeezing the calf muscles, the tendon is likely intact. If there is no movement of the foot, the Achilles tendon has likely been torn. If a patient experiences sharp calf muscle pain upon dorsiflexion of his or her foot, Homan's sign is said to be present; this finding indicates a deep venous thrombosis. A Babinski response is present if the patient's big toe moves upward when you stroke the sole of the foot with a blunt object. A present Babinski response is normal in infants; however, it is a sign of nervous system injury if it is present in older children and adults. The term "foot drop" is used to describe a neuromuscular disorder that affects the patient's ability to dorsiflex the foot. Foot drop is often caused by injury to the peroneal nerve - a division of the sciatic nerve that runs along the outside of the lower leg and branches off into each ankle, foot, and first two toes. It transmits signals to muscle groups responsible for ankle, foot, and toe movement and sensation.

Which of the following clinical presentations is MOST consistent with exposure to chemicals such as tabun, sarin, and soman? A) Hypertension, dysphagia, bronchospasm, blistering B) Photophobia, abdominal pain, tachycardia, headache C) Hematemesis, seizures, diplopia, spontaneous bleeding D) Rhinorrhea, nausea and vomiting, bradycardia, polyuria

D. Tabun, sarin, and soman are chemical nerve agents; they are in a class of chemicals called organophosphates. Such chemicals inhibit the effects of acetylcholinesterase (AChE) - the chemical mediator of acetylcholine (ACh). ACh is the chemical neurotransmitter of the parasympathetic nervous system. By inhibiting the AChE enzyme from breaking down ACh, both the levels and duration of action of ACh increase; this results in severe parasympathetic nervous system stimulation. In addition to severe bradycardia and hypotension, AChE inhibitor exposure produces signs and symptoms that can be remembered with the mnemonics DUMBELS or SLUDGEM. DUMBELS stands for Defecation, Urination, Miosis (pupillary constriction), Bradycardia/Bronchorrhea, Emesis, Lacrimation, and Salivation. SLUDGEM stands for Salivation, Lacrimation, Urination, Defecation, Gastrointestinal distress, Emesis, and Miosis.

You are assessing a patient with an injury to the left midshaft femur. Which of the following is the LEAST reliable indicator of an underlying fracture? A) Localized pain B) Gross deformity C) Shortening of the extremity D) Ecchymosis and swelling

Ecchymosis (bruising) and swelling are common clinical signs associated with any type of musculoskeletal injury - fractures, dislocations, fracture/dislocations, and sprains. In particular, swelling often obscures underlying deformity - one of the most reliable signs of a fracture. The most reliable symptom of an underlying fracture is pain that is well localized to the fracture site. Shortening occurs in fractures when the broken ends of a bone override one another; this is characteristic of femur fractures, for example, because the fractured femur can no longer serve as a strut to oppose spasm in the powerful thigh muscles.

After placing an endotracheal tube in a cardiac arrest patient, large amounts of vomitus immediately begin flowing out of the tube. You should: A) withdraw the ET tube just until the flow of vomitus stops and inflate the distal cuff with 5 to 10 mL of air. B) immediately remove the ET tube, resume bag-mask ventilations, and reattempt intubation after 2 minutes. C) leave the ET tube in place, fold it to the side so the vomitus can drain, and resume bag-mask ventilations. D) apply posterior pressure to the cricoid cartilage to compress the esophagus and stop the flow of vomitus.

If inadvertent esophageal intubation occurs and vomitus begins flowing out of the ET tube, you should leave the ET tube in place, inflate the distal cuff with more air than normal (perhaps as much as 20 to 30 mL), fold the ET tube to the side to allow the vomitus to drain, and resume bag-mask ventilations. If you remove the ET tube when the patient is regurgitating, then you have virtually assured aspiration. With proper technique, you can maintain an adequate mask seal over the folded ET tube in order to ventilate the patient with the bag valve mask. Applying posterior cricoid pressure may compress the esophagus, but will NOT compress the ET tube that is in the esophagus. Consider passing a gastric tube through the ET tube that is in the esophagus and connect it to suction; doing so will decompress the stomach.

You are assessing the cardiac rhythm of a woman with respiratory distress. The rhythm is irregularly irregular with a rate of 120 beats/min. The QRS complexes measure 0.10 seconds in duration, the P wave to QRS ratio is 1:1, and the P waves vary in shape. Which of the following BEST fits this description? A) Atrial fibrillation B) Wandering atrial pacemaker C) Atrial flutter with aberrancy D) Multifocal atrial tachycardia

In multifocal atrial tachycardia (MAT), the pacemaker of the heart moves within various areas of the atria. MAT is characterized by a ventricular rate that is greater than 100 beats/min. MAT is irregularly irregular, with variation between R-R intervals based on the site of the pacemaker for that particular complex. P waves are present, upright, and precede each QRS complex; however, the shapes of the P waves vary as an indication of their different sites of origin. The P-R interval generally measures between 0.12 and 0.20 seconds, but also varies slightly based on the origin of the particular complex. Atrial fibrillation (A-Fib) is also an irregularly irregular rhythm; however, there are no discernable P waves. A wandering atrial pacemaker essentially contains all the components of MAT; unlike MAT, however, the ventricular rate is typically less than 100 beats/min. Atrial flutter (A-Flutter) has characteristic flutter waves (F waves) that resemble a saw tooth. If accompanied by aberrancy, A-flutter has QRS complexes that are greater than 0.12 seconds in duration, which indicates abnormal (aberrant) ventricular conduction.

Mastoid bruising and cerebrospinal otorrhea following diffuse impact to the head are MOST indicative of: A) a basilar skull fracture. B) massive cerebral edema. C) herniation of the brainstem. D) a fractured cribriform plate.

Mastoid bruising (Battle's sign) and cerebrospinal otorrhea (cerebrospinal fluid [CSF] leakage from the ears) are classic signs of a basilar skull fracture. Basilar skull fractures are associated with high-energy trauma, and usually occur following diffuse impact to the head (eg, falls, motor-vehicle crashes). A basilar skull fracture generally results from extension of a linear fracture to the base of the skull. Cerebrospinal rhinorrhea (CSF draining from the nose) suggests a fractured cribriform plate. The cribriform plate of the ethmoid bone is a horizontal bone that is perforated with numerous foramina (openings) that allow passage of the olfactory nerve filaments from the nasal cavity. Cerebral edema (swelling of the brain) can result from any significant head injury; it is not exclusive to a basilar skull fracture. Signs of brainstem herniation - which represent significant intracranial pressure - include abnormal respiratory patterns (ie, Biot's respirations, ataxic respirations), posturing (flexor or extensor), and pupillary abnormalities (asymmetric or fixed and dilated pupils).

A trauma patient is being transferred from one facility to another. The patient is intubated and is on a mechanical transport ventilator. During transport, the high-pressure alarm sounds on the ventilator. Which of the following would MOST likely cause this? A) The cuff on the ET tube has ruptured B) The source oxygen cylinder is below 500 psi C) The ET tube is in the right mainstem bronchus D) The ET tube is displaced and is in the esophagus

Mechanical transport ventilators are commonly used in the prehospital setting; therefore, the paramedic should possess basic troubleshooting skills for dealing with ventilator malfunction and alarms. A high-pressure alarm indicates that air is meeting resistance somewhere in the ventilator circuit. Causes of high-pressure alarms include obstructed or kinked ventilator tubing, a developing pneumothorax, or migration of the ET tube into a mainstem bronchus. The ventilator is calibrated to ventilate both lungs; if all of this volume is suddenly directed into one lung, the high-pressure alarm will sound. A low oxygen cylinder pressure would not cause a high-pressure alarm. A low-pressure alarm could indicate rupture of the ET tube cuff, a leak somewhere in the ventilator circuit, or displacement of the ET tube in the esophagus. If the ET tube suddenly moves from an area of higher pressure (the lungs) to an area of lower pressure (the stomach), the low-pressure alarm will sound.

A 6-month-old infant with bacterial meningitis would MOST likely present with: A) nuchal rigidity, a sunken fontanelle, and fever. B) irritability, generalized urticaria, and vomiting. C) fever, paradoxical irritability, and poor feeding. D) hyperglycemia, nuchal rigidity, and low-grade fever.

Meningitis, inflammation or infection of the protective covering of the brain and spinal cord (meninges), is usually caused by a viral or bacterial infection. Viral meningitis is rarely life-threatening; however, bacterial meningitis is potentially fatal. In the early stages of the illness, it can be difficult to distinguish viral from bacterial meningitis, so always proceed as if the child has bacterial meningitis. The child's presentation varies depending on his or her age and the type of infection present. The younger the child, the more vague the symptoms. Infants often have a fever (usually high-grade), poor feeding, a bulging fontanelle, vomiting, and paradoxical irritability. A child with paradoxical irritability becomes more irritable or cries when picked up by a caregiver due to pain caused by aggravation of an already inflamed spinal cord. Unlike older children, infants and small children rarely present with nuchal rigidity (neck pain or stiffness), mainly because they don't have much of a neck. Pneumococcal and meningococcal meningitis account for most cases in children. Meningococcal meningitis may cause seizures, septic shock, and death, and is often accompanied by a petechial (small, pinpoint red spots) or purpuric (large purple or black spots) rash, in addition to other signs and symptoms of meningitis. Urticaria (hives) is associated with allergic reactions, not meningitis. Due to the hypermetabolic state that occurs when children are attempting to compensate for shock, they can deplete their glucose stores, resulting in hypoglycemia.

You are assessing the cardiac rhythm of a 9-year-old child who says his chest is "fluttering." P waves are not visible, the QRS complexes measure 70 ms in duration, the heart rate is 200 beats/min, and there is no variation in the R-R intervals when the child moves. These findings are MOST consistent with: A) reentry supraventricular tachycardia. B) ventricular tachycardia. C) ectopic atrial tachycardia. D) sinus tachycardia.

Most narrow-complex tachycardias in infants and children are sinus tachycardia. However, some children present with reentry supraventricular tachycardia (SVT). In children older than 1 year of age, reentry SVT is characterized by a ventricular rate greater than 180 beats/min (> 220 beats/min in infants), narrow (less than or equal 90 ms) QRS complexes, and no variability in the R-R intervals with movement or activity. Furthermore, children with reentry SVT typically do not present with a history that is compatible with common causes of sinus tachycardia (ie, fever, hypovolemia); in some cases, the child may have a history of reentry SVT. In sinus tachycardia, the heart rate in children is usually less than 180 beats/min (< 220 beats/min in infants), the R-R intervals vary with activity or movement, and the child has a history that is compatible with common causes of sinus tachycardia (ie, fever, hypovolemia). Ventricular tachycardia (V-Tach) is uncommon in children, but should be suspected if the QRS complexes measure greater than 90 ms in duration. Ectopic atrial tachycardia (EAT), also uncommon in children, is characterized by P waves of varying morphologies. Diagnosing EAT in children with tachycardia is often not possible, however, because the P waves are usually not visible.

A young male intentionally ingested unknown quantities of ibuprofen, aspirin, codeine, and Anafranil. He is unresponsive, and is hypoventilating, bradycardic, and hypotensive. Which of these drugs is the MOST likely cause of his clinical presentation? A) Ibuprofen B) Aspirin C) Codeine D) Anafranil

Of the drugs listed, codeine is the most likely cause of this patient's clinical presentation. Codeine is a narcotic analgesic, and when taken in excess, it can result in central nervous system depression (ie, altered mental status, hypoventilation, bradycardia, hypotension). Aspirin toxicity causes metabolic acidosis and would likely manifest with hyperventilation, an indication that the respiratory buffer system is attempting to eliminate acids from the body. Clomipramine (Anafranil) is a tricyclic antidepressant and would likely result in tachycardia when taken in excess. Ibuprofen, the active ingredient in Advil and Motrin, erodes the gastric lining and can be toxic to the liver; central nervous system depression is not indicative of ibuprofen toxicity.

You and your partner are triaging four patients at the scene of a motor-vehicle crash while awaiting the arrival of additional ambulances. Which of the following patients would be triaged as delayed? A) Anterior neck pain; stridorous breathing; history of type 2 diabetes B) Tibial fracture; hematoma to the forehead; conscious but confused C) Bilaterally deformed femurs; severe neck pain; conscious and alert D) Abdominal distention; pallor; responds appropriately to questions

Of the injuries and conditions listed, the conscious and alert patient who has bilaterally deformed femurs and severe neck pain appears to be the least critical and would therefore be triaged as delayed (yellow tag). Patients should be triaged as immediate (red tag) if they have airway or breathing problems, severe or uncontrolled bleeding, altered mental status, signs of shock, severe underlying medical problems, and open chest or abdominal injuries. All of the other patients listed have injuries or conditions that place them in an immediate triage category. Assuming an adequate mental status, a patent airway, no evidence of respiratory or circulatory compromise, and no signs of shock, patients with major or multiple bone or joint injuries and neck or back injuries would be triaged as delayed.

Which of the following combinations of drugs are indicated for a patient with copious bronchial secretions, marked bradycardia, and profuse diaphoresis following exposure to an industrial pesticide? A) Atropine and physostigmine B) Epinephrine and 2-PAM chloride C) Pralidoxime chloride and diazepam D) Atropine and pralidoxime chloride

Organophosphate and carbamate pesticides are cholinergic agents that stimulate the parasympathetic nervous system (PNS) by deactivating acetylcholinesterase (AChE). AChE is an enzyme that regulates the degradation of acetylcholine (ACh), the chemical neurotransmitter of the PNS. Without AChE, ACh accumulates and causes profound stimulation of the PNS. The mnemonic DUMBELS is useful for remembering the signs and symptoms of organophosphate and carbamate poisoning; it stands for defecation, urination, miosis (pupillary constriction), bradycardia and bronchorrhea, emesis, lacrimation, and salivation. Atropine sulfate - an anticholinergic agent - is the first-line drug used in the treatment of organophosphate/carbamate poisoning. Atropine competitively antagonizes the action of ACh and reverses its muscarinic effects (ie, bradycardia, excessive secretions). Atropine is given in a dose of 1 to 2 mg every 5 to 15 minutes until secretions clear and the heart rate increases; higher doses are often required for patients with severe toxicity. Pralidoxime chloride (2-PAM chloride, Protopam) is given after atropine to reactivate AChE, allowing it to resume its normal function. The dose is 600 mg via autoinjector or 1 to 2 g IV over 15 to 30 minutes. Atropine and pralidoxime are commonly given together via autoinjector (Duodote). Seizures are common in patients with organophosphate/carbamate poisoning, and should be terminated with a benzodiazepine (ie, lorazepam [Ativan], diazepam [Valium]). Physostigmine is an AChE inhibitor that interferes with the metabolism of ACh. Because it inhibits the action of AChE, physostigmine is clearly not indicated and would only exacerbate the patient's condition.

During your SAMPLE history of an elderly man, he tells you that his cardiologist told him that he has an "irregular heartbeat." His medications include warfarin sodium and digoxin. On the basis of this information, what underlying cardiac rhythm should you suspect? A) AV heart block B) Atrial fibrillation C) Atrial tachycardia D) Sinus dysrhythmia

Patients with atrial fibrillation (A-Fib) are commonly prescribed digoxin (a digitalis preparation) and warfarin sodium (Coumadin). Digoxin is a positive inotrope and a negative chronotrope; it is a useful drug to treat congestive heart failure and to control the ventricular rate of atrial fibrillation or atrial flutter. Warfarin is an anticoagulant medication. As the atria fibrillate, blood can stagnate and form microemboli that can be ejected from the heart and occlude a pulmonary, cerebral, or coronary artery.

A hiker was bitten on the leg by a rattlesnake approximately 20 minutes ago. He complains of pain to the lateral aspect of his left leg, just proximal to the ankle. Assessment of that area reveals two small puncture wounds surrounded by edema. You also note localized twitching of his calf muscles. He is conscious and alert, but anxious. His blood pressure is 112/70 mm Hg, pulse is 120 beats/min, and respirations are 24 breaths/min with adequate depth. In addition to keeping him calm, you should: A) administer high-flow oxygen via nonrebreathing mask, apply a tight constricting band proximal to the wound, cover the wound with a sterile dressing, start an IV line and set it at a KVO rate, and transport. B) place him in a supine position, ventilate him with a bag-valve-mask device, immobilize his leg and elevate it 6 to 12 inches, start an IV line and give a 20-mL/kg crystalloid bolus, and transport. C) apply a chemical icepack to the wound to decrease venom absorption, immobilize his leg and keep it below the level of the heart, administer high-flow oxygen via nonrebreathing mask, and transport. D) administer high-flow oxygen via nonrebreathing mask, apply a sterile dressing to the wound, immobilize his leg and keep it below the level of the heart, start an IV line and set it at a KVO rate, and transport.

Pit viper venom contains a mix of enzymes that cause local tissue necrosis, hemolysis, increased vascular permeability, coagulopathy, and neuromuscular dysfunction. Local signs of envenomation include puncture marks, pain, edema, and fasciculations (small, local, involuntary muscle twitching). Systemic signs include nausea and vomiting, dizziness, weakness, numbness or tingling of the mouth and tongue, tachycardia, hypotension, and tachypnea. After ensuring scene safety, ensure that the patient has a patent airway and keep him calm to decrease venom spread and absorption. Your patient, although tachypneic, has adequate tidal volume. Give high-flow oxygen via nonrebreathing mask, but be ready to assist his ventilations. Establish vascular access and set the flow rate to keep the vein open; monitor his BP and give fluid boluses as needed. Cover the wound with a sterile dressing, immobilize his leg, and keep it below the level of the heart. Do NOT apply ice to a snakebite wound; this causes vasoconstriction and may increase venom absorption. Transport to a facility where he can receive antivenin (eg, CroFab).

Polycythemia is a condition that results in: A) lactic acidosis due to anaerobic metabolism. B) a marked increase in the core body temperature. C) increased oxygen-carrying capacity of the blood. D) spontaneous bleeding due to a low platelet count.

Polycythemia is defined as an increase in red blood cell production. It often occurs in response to hypoxia, but may occur for other reasons. Patients with emphysema, for example, are often polycythemic in response to chronic hypoxia; this is why they are commonly referred to as "pink puffers." Because red blood cells attach to the hemoglobin molecule and carry oxygen, polycythemia increases the oxygen-carrying capacity and efficiency of the blood. Of course, how well the tissues actually get oxygenated depends on the oxyhemoglobin saturation. A marked increase in core body temperature results from environment factors (eg, heatstroke), inflammation, and infection. Pyrogens are fever-causing agents that are produced by the immune system. When an infectious or inflammatory process exists, the body produces excess levels of pyrogens, resulting in pyrexia (fever). Patients with thrombocytopenia - a reduction in platelets - commonly experience spontaneous bleeding. Furthermore, thrombocytopenic patients have impaired hemostasis; even minor internal bleeding can be extremely serious. Aerobic metabolism is the normal metabolic process; it produces carbon dioxide and water as its byproducts. In the absence of oxygen, the cells convert from aerobic to anaerobic metabolism and produce lactic acid.

Which of the following MOST accurately describes the process of gas exchange in the lungs? A) The transfer of carbon dioxide from the alveoli into the bloodstream is facilitated by a process called diffusion. B) The gases exchanged in the lungs move from an area of greater concentration to an area of lesser concentration. C) Blood that returns to the lungs from the right side of the heart has a slightly lower level of carbon dioxide than oxygen. D) The partial pressure of oxygen in the alveoli is typically between 40 and 50 torr at the end of a maximal inhalation.

Pulmonary (external) respiration is defined as the exchange of gases in the lungs. Gases exchanged in the lungs (O2 and CO2) move from an area of greater concentration to an area of lesser concentration by a process called diffusion. Blood that enters the lungs from the right side of the heart has a PO2 of approximately 40 mm Hg and a PCO2 of approximately 46 mm Hg. Within the lungs, carbon dioxide diffuses from the bloodstream into the alveoli while oxygen diffuses from the alveoli into the bloodstream. The partial pressure of oxygen within the alveoli is near 100 mm Hg, while the partial pressure of carbon dioxide is near 0 mm Hg.

In which of the following conditions would you MOST likely detect a drop in systolic blood pressure during inhalation? A) Severe asthma B) Chronic bronchitis C) Pulmonary edema D) Simple pneumothorax

Pulsus paradoxus, a significant drop in the systolic blood pressure during inhalation, indicates physical restriction of cardiac movement during inhalation. Pulsus paradoxus can also be characterized by a marked weakening (or even disappearance) of the pulse during inhalation. Pulsus paradoxus may be observed in patients experiencing a severe asthma attack or an exacerbation of emphysema, in which case air-trapping in the lungs leads to pulmonary hyperinflation. As the hyperinflated lungs put pressure against the heart during inhalation, cardiac movement can be physically restricted. Pulsus paradoxus may also be observed in patients with a severe pericardial tamponade.

You are assessing a 29-year-old male with an apparent emotional crisis. He is agitated and refuses to give you any information because his roommate told him not to trust anyone. His small apartment clearly indicates that he is the sole occupant. Your partner finds a prescription for Zyprexa, which is prescribed to the patient. This patient MOST likely has: A) schizophrenia. B) bipolar disorder. C) trichotillomania. D) obsessive-compulsive disorder.

Schizophrenia is a psychiatric illness marked by impairments in the perception or expression of reality. Signs include auditory hallucinations (eg, the "roommate"), paranoid or bizarre delusions (eg, you are not to be trusted), and disorganized speech and thinking. Antipsychotics such as Olanzapine (Zyprexa), risperidone (Risperdal) and clozapine (Clozaril) are commonly prescribed to treat schizophrenia. The patient's presentation is not consistent with bipolar disorder. Bipolar disorder is a biphasic psychiatric condition characterized by alternations of depression and mania. Manic patients are excessively elated, overly talkative, and have accelerated speech. Medications used to treat bipolar disorder include lithium, quetiapine fumarate (Seroquel) and aripiprazole (Abilify). Trichotillomania, the recurrent pulling out of one's own hair, is an impulse control disorder. Nothing in the scenario suggests this condition. Obsessive-compulsive disorder (OCD) is a psychiatric condition characterized by intrusive and unwanted thoughts or worries (obsessions), which the patient attempts to alleviate or eliminate by performing ritualistic acts (compulsions). Selective serotonin reuptake inhibitor (SSRI) drugs, such as fluoxetine (Prozac), sertraline (Zoloft), and paroxetine (Paxil), are commonly prescribed to patients with OCD. This patient's behavior is not consistent with OCD.

Which of the following statements regarding sickle cell disease is correct? A) In sickle cell disease, misshapen red blood cells can lodge in the spleen, causing it to swell and rupture. B) When the sickle cell gene is inherited from both parents, the patient will probably not develop the disease. C) The red blood cells in sickle cell disease have a rounded appearance, rather than the normal oblong appearance. D) Despite their altered shape, the red blood cells of a person with sickle cell disease are effective carriers of oxygen.

Sickle cell disease is the most commonly inherited blood disorder. Although it primarily affects African-American, Puerto Rican, and European populations, it can occur in anyone. Sickle cell disease starts with a gene defect of the adult-type hemoglobin (HbA). This mutated gene can be inherited from one parent (HbS) or both parents (HbSS). When the gene is inherited from both parents, there is a high probability that the patient will be prone to sickle cells (that is, the person actually has the disease) or the sickle cell trait (the person is a carrier of the mutated gene). The defective red blood cells (RBCs) are misshapen; affected cells have an oblong (sickle) shape instead of a smooth, round shape. This shape makes the RBCs poor carriers of oxygen, which means that a person with sickle cell disease is highly susceptible to hypoxia. The odd shape may also cause RBCs to lodge in small blood vessels (thrombotic crisis) or in the spleen (splenic sequestration crisis), causing the organ to swell and rupture, which can cause death. Sickle cell disease may also lead to other crises, such as aplastic crisis and hemolytic crisis. In aplastic crisis, RBC production temporarily stops. In hemolytic crisis, the RBCs break down quickly. In acute crises, patients often have severe pain, which will require aggressive pain management. They may also experience frequent infections, which can lead to sepsis and death.

An older woman presents with severe weakness, hypotension, lower back pain, and vomiting. Her husband tells you that she has not taken her prednisone in several days because she has not been feeling well. Which of the following should you suspect? A) Addisonian crisis B) Cushing syndrome C) Pheochromocytoma D) Thyrotoxic crisis

Signs and symptoms of acute adrenal insufficiency can manifest suddenly in what is called an addisonian crisis. Abrupt cessation of corticosteroid therapy (ie, prednisone, hydrocortisone) is the most common cause of an addisonian crisis. It may also be triggered by acute exacerbation of chronic adrenal insufficiency (Addison disease), usually brought on by stress, trauma, surgery, or a severe infection. In either case, cardiovascular collapse occurs due to a lack of the hormone cortisol; therefore, the chief clinical manifestation of an addisonian crisis is shock. Other signs and symptoms may include weakness; lethargy; fever; severe pain in the lower back, legs, or abdomen; and severe vomiting and diarrhea. Cushing syndrome is caused by excessive cortisol production by the adrenal cortex; it may also occur if large amounts of corticosteroids are administered. Pheochromocytoma is an adrenal tumor that causes excessive release of epinephrine and norepinephrine; patients with this condition present with hypertension and tachycardia. Thyrotoxic crisis (thyroid storm) is a condition caused by critically high thyroid hormone levels, resulting in a hypermetabolic state. Signs and symptoms include severe tachycardia, hypertension, fever, altered mental status, and possibly heart failure.

A 9-year-old child has generalized weakness; blood in the stool; and bruising, even from minor trauma. These findings are MOST consistent with: A) anemia. B) lymphoma. C) leukemia. D) sickle cell crisis.

Signs of leukemia (cancer of the blood) include fatigue or weakness, easy bruising, and spontaneous bleeding. A patient with anemia can have similar signs and symptoms, but tends to have cutaneous bleeding and an unusual craving for ice. Lymphoma, cancer of the lymphatic system, is characterized by weakness, enlarged lymph nodes (lymphadenopathy), fever, weight loss, and anorexia, among others. Signs and symptoms of sickle cell crisis depend on the body system affected, and include severe joint or bone pain, abdominal pain (from an enlarged spleen [splenomegaly]), and chest pain and hemoptysis (coughing up blood).

You are dispatched to a residence for a patient having a seizure. Upon arriving at the scene, you find that the patient, a 39-year-old male, is experiencing a generalized tonic-clonic seizure and is cyanotic. His wife tells you that he has been like this for the past 25 minutes. You should: A) ventilate him with a bag-valve-mask device, establish IV access, and administer 5 mg of diazepam. B) give high-flow oxygen via nonrebreathing mask, place him on his side, and wait for the seizure to stop. C) administer 0.5 mg/kg of diazepam rectally, cover him with a blanket, and administer high-flow oxygen. D) intubate him to prevent aspiration, establish IV access, and administer 0.1 mg/kg of lorazepam.

Status epilepticus is defined as a prolonged (> 10 minutes) seizure or consecutive seizures that occur without an intervening return to consciousness. During a seizure, neurons are using huge amounts of glucose and producing lactic acid. For short periods, this does not cause long-term damage. If the seizure continues, however, the body can't remove the waste products effectively or ensure adequate glucose supplies; this can cause neuronal damage or death. Hypoventilation or apnea is common during a seizure; cyanosis is also common. In short-duration seizures, these are usually short-lived (< 30 seconds) and generally do not require assistance. However, status epilepticus causes prolonged hypoventilation or apnea and requires ventilation assistance, although this may be difficult. Patients experiencing a seizure often have clenched teeth (trismus); therefore, intubation will be nearly impossible and is generally contraindicated. Benzodiazepines (ie, Valium, Ativan) are used to terminate seizures. The dose of Valium is 5 mg IV, IO, or IM (repeated every 10 to 15 minutes up to 30 mg). If vascular access is not available, Valium can be given rectally (adult dose, 0.2 mg/kg). The dose of Ativan is 0.05 mg/kg (maximum single dose of 4 mg); this may be repeated in 10 to 15 minutes if needed. If benzodiazepines do not terminate the seizure and the patient cannot be ventilated, proceed with rapid sequence induction (RSI) and intubation.

When assessing lead II in a patient with a heart rate of 70 beats/min, the Q-T interval is considered prolonged if it is: A) twice the width of the QRS complex. B) consistently greater than 0.20 seconds. C) greater than one half of the R-R interval. D) three times the length of the P-R interval.

The Q-T interval represents the time from the beginning of ventricular depolarization to the end of ventricular repolarization, and is measured from the start of the QRS complex to the end of the T wave. In a patient with a heart rate between 60 and 100 beats/min, the Q-T interval in lead II is considered to be prolonged if it is greater than one half the distance between any two R waves (R-R interval). If the Q-T interval is prolonged, the patient is at increased risk for developing a lethal dysrhythmia; an electrical impulse may fire during the relative refractory period (downslope of the T-wave), resulting in monomorphic or polymorphic ventricular tachycardia (with or without a pulse) or ventricular fibrillation. If lead II suggests Q-T prolongation, a 12-lead ECG should be obtained to quantify this finding. In a normocardic patient (heart rate of 60 to 100 beats/min), the corrected Q-T interval (QTc) should range between 0.36 and 0.44 seconds (360 to 440 milliseconds) on the 12-lead ECG. The Q-T interval is corrected based on the patient's heart rate. The faster the heart rate, the narrower the Q-T interval; the slower the heart rate, the wider the Q-T interval.

In older adults, an S3 heart sound: A) is generally very pronounced. B) indicates mitral valve closure. C) signifies moderate heart failure. D) is considered a normal variant.

The S3 or third heart sound is a soft, low-pitched sound that is caused by vibrations of the ventricular walls, resulting from the rapid filling period of the ventricle during the beginning of diastole. An S3 sound should occur 120 to 170 milliseconds after S2, if it is heard at all. An S3 sound may be a normal clinical finding in children and young adults, although a cardiac evaluation should be performed to determine this. When it is heard in older adults, however, it signifies moderate to severe heart failure. S1 is heard near the beginning of ventricular contraction (systole), when the tricuspid and mitral valves close. S2 is heard near the beginning of ventricular relaxation (diastole), when the pulmonic and aortic valves close.

In an otherwise healthy 70-kg adult, hypotension is typically noted after he or she has lost ____________ of his or her blood volume. A) at least 10% to 15% B) more than 1,500 mL C) between 15% and 20% D) more than 700 to 800 mL

The average adult has a total blood volume (TBV) of about 5 liters. By assessing certain hemodynamic parameters, you can estimate the percentage of blood loss, and thus, the severity of hemorrhagic shock. Hemorrhagic shock is categorized into four classes. A healthy adult can safely tolerate up to 15% (750 mL) TBV loss (Class I). This volume of blood loss falls well within the body's ability to compensate. Loss of 15% to 30% (750 to 1,500 mL) of a person's TBV (Class II) requires more sophisticated compensation; it is characterized by tachycardia (> 100 beats/min), tachypnea (20 to 30 breaths/min), mild anxiety, and a narrowing pulse pressure. The BP, however, is still maintained; thus, the patient is said to be in compensated shock. Hypotension usually appears after the patient has lost more than 30% (1,500 mL) of his or her TBV (Class III). Compensatory mechanisms are failing and the BP can no longer be maintained (decompensated shock). The patient has more pronounced tachycardia (> 120 beats/min) and tachypnea (30 to 40 breaths/min), narrowed pulse pressure, and marked anxiety or confusion. Survival is possible with early recognition and rapid transport to a trauma center. Class IV hemorrhage occurs when the patient loses more than 40% (2,000 mL) of his or her TBV. The patient is markedly tachycardic (> 140 beats/min), tachypneic (> 40 breaths/min), and hypotensive, and is usually lethargic or comatose. Many patients with Class IV hemorrhage die in the prehospital setting because immediate definitive care is not available. For this reason, they are often said to be in irreversible shock.

You are assessing the cardiac rhythm of a 6-year-old child. The rhythm is regular, the rate is 170 beats/min, the QRS complexes measure 0.11 seconds, and P waves are not visible. You should suspect: A) atrial fibrillation. B) sinus tachycardia. C) ventricular tachycardia. D) supraventricular tachycardia.

The cardiac rhythm described fits the definition of ventricular tachycardia (V-Tach). V-Tach in children is characterized by a regular rhythm, a rapid rate, QRS complexes that are greater than 0.09 seconds in duration, and absent P waves. Supraventricular tachycardia (SVT) in children is characterized by a regular rhythm, a rate greater than 180 beats/min (> 220 beats/min in infants), QRS complexes that are equal to or less than 0.09 seconds in duration, and absent P waves (P waves may be present, but the ventricular rate is too fast for them to be visible). In SVT, there is no beat-to-beat variability during patient movement. Sinus tachycardia in children is characterized by a regular rhythm, a rate less than 180 beats/min (< 220 beats/min in infants), QRS complexes that are equal to or less than 0.09 seconds in duration, and visible P waves (the rate can often bury the P waves). In sinus tachycardia, beat-to-beat variability is noted during patient movement. Atrial fibrillation in children is characterized by an irregularly irregular rhythm, a variable rate, QRS complexes that are less than or equal to 0.09 seconds in duration, and absent P waves.

A 4-year-old male presents with audible stridor, a barking cough, and increased work of breathing. He is conscious and alert; has pink, warm skin; and has a heart rate of 120 beats/min. Further assessment reveals clear and equal lung sounds bilaterally, an oxygen saturation of 97%, and a temperature of 99.2° F. You should: A) give him 10 mg/kg of pediatric acetaminophen, administer high-flow oxygen via nonrebreathing mask, and establish vascular access. B) administer a 0.5-mg unit dose of ipratropium via nebulizer, give him oxygen as tolerated, and make preparations to perform tracheal intubation. C) let him assume a position of comfort, offer oxygen via the blow-by technique, and administer a 2.25% solution of racemic epinephrine via nebulizer. D) avoid agitating him, establish vascular access and set the rate to keep the vein open, and give 2.5 to 5 mg of albuterol via nebulizer up to three times.

The child symptoms are consistent with croup (laryngotracheobronchitis), a viral upper airway infection. Parainfluenza virus causes most cases of croup, although respiratory syncytial virus (RSV) and adenovirus have been implicated. Croup typically affects children between 6 months and 6 years of age, usually during fall and winter. Croup causes swelling of the larynx and trachea, resulting in a barking, seal-like cough. Most cases of croup are mild and do not cause respiratory failure. The child is typically alert, has a barking cough, increased work of breathing, stridor with agitation, and normal skin color. Breath sounds are usually clear and equal bilaterally. Allow the child to assume a position of comfort and avoid agitating him. Your patient has respiratory distress, but no overt signs of hypoxia (eg, low Sp02, bradycardia, cyanosis); therefore, assisted ventilation and intubation are not indicated. Give oxygen via the blow-by technique, if tolerated. Consider giving nebulized racemic epinephrine (0.5 mL of a 2.25% solution in 3 mL of normal saline). Epinephrine reduces upper airway edema through vasoconstriction. Croup is an upper airway problem and does not cause wheezing; therefore, bronchodilators (eg, Ventolin, Atrovent) will be of no real benefit. IV access should be deferred; the child is stable and an IV will agitate him, potentially worsening his condition. His temperature does not warrant treatment with Motrin or Tylenol. Transport the child and monitor him en route.

Increased parasympathetic tone, bradycardia, shunting of blood to the brain, and hypotension describes: A) Cushing's reflex. B) the diving reflex. C) Beck's triad. D) Cullen's sign.

The diving reflex, also known as the mammalian diving reflex, is a protective mechanism of the body that is most prominent in cold temperatures (ie, falling in cold water). Through increased parasympathetic tone, the pulse rate and blood pressure both fall to decrease overall oxygen demand and consumption, while at the same time, blood is shunted to the brain to sustain it for as long as possible. The diving reflex is the reason why small children are able to survive for extended periods of time when submerged in cold water. The effect of the diving reflex diminishes with age. Cushing's reflex, also called Cushing's triad, is a trio of clinical signs in patients with increased intracranial pressure; it includes hypertension, bradycardia, and abnormal breathing. Cullen's sign, bruising around the umbilicus, is an indicator of blood in the peritoneal space. Beck's triad, a trio of clinical signs observed in patients with a severe pericardial tamponade, includes muffled or distant heart tones, jugular venous distention, and a narrowing pulse pressure.

What is the fraction of inspired oxygen (FiO2) delivered to an apneic patient who is receiving bag-mask ventilation without supplemental oxygen attached? A) 0.05 B) 0.16 C) 0.21 D) 0.55

The fraction of inspired oxygen (FiO2) is the percentage of oxygen that a patient is receiving, either during normal breathing or during assisted breathing; it is documented as a decimal (ie, 1.0 equals 100%; 0.9 equals 90%, etc). A bag-valve-mask device not attached to supplemental oxygen will deliver room air oxygen, which is 21% (0.21). Mouth-to-mask ventilation delivers an FiO2 of 16% (0.16), since the rescuer is exhaling into the patient (exhaled air contains 16% oxygen). If oxygen is attached to a pocket mask, an FiO2 of up to 55% (0.55) can be delivered.

Which of the following organs would produce the MOST severe blood loss following trauma to the abdomen? A) Liver B) Spleen C) Kidney D) Pancreas

The liver is a very large, highly vascular organ that contains a large volume of blood at any given time. It produces several blood coagulation factors, including fibrinogen (I), prothrombin (II), ionized calcium (IV), labile factor/proaccelerin (V), VI, and stable factor/proconvertin (VII). If the liver is damaged or diseased, it can take longer for the body to form clots. This would result in increased bleeding time and a large volume of blood loss.

A 30-year-old unrestrained woman struck the steering wheel when her car hit a tree while traveling at 40 miles per hour. She reports pain to the midsternal area, which is point tender to palpation. Her blood pressure is 100/60 mm Hg, pulse is 118 beats/min and irregular, and respirations are 26 breaths/min and shallow. The remainder of your assessment is unremarkable. You should suspect: A) a flail chest. B) pericardial tamponade. C) myocardial contusion. D) esophageal injury.

The mechanism of and location of the injury, as well as the irregularity of the patient's pulse, are suggestive of a myocardial contusion. Patients with this type of injury can experience all of the same deleterious effects associated with an acute myocardial infarction, including cardiogenic shock, arrhythmias, and cardiac arrest. Although a flail chest, pericardial tamponade, or esophageal injury cannot be completely ruled out in the field, there are no physical exam findings that suggest any of these injuries.

A 40-year-old patient has a blood glucose level of 800 mg/dL and is breathing deeply at a rate of 34 breaths/min. Which of the following should the paramedic expect to observe? A) Small capnographic waveforms; low end-tidal CO2 reading B) Large capnographic waveforms; low end-tidal CO2 reading C) Small capnographic waveforms; high end-tidal CO2 reading D) Large capnographic waveforms; high end-tidal CO2 reading

The paramedic should suspect that the patient has an underlying metabolic acidosis secondary to ketoacidosis. The patient's breathing pattern is consistent with Kussmaul respirations, which is the respiratory system's attempt to create a compensatory respiratory alkalosis. Hyperventilation drives the body's carbon dioxide level down; this would cause a low (< 35 mm Hg) end-tidal CO2 value and small capnographic waveforms. High (> 45 mm Hg) end-tidal CO2 values and large capnographic waveforms are consistent with CO2 retention, such as what you would expect when a patient is hypoventilating (ie, opiate overdose, respiratory failure).

A 54-year-old man presents with chest pressure, confusion, and profuse diaphoresis. As your partner administers supplemental oxygen, you apply the cardiac monitor. In lead II, you observe a wide QRS complex rhythm with dissociated P waves and a ventricular rate of 35 beats/min. You should: A) immediately obtain a 12-lead ECG. B) obtain a complete set of vital signs. C) begin transcutaneous pacing at once. D) start an IV and give 0.5 mg of atropine.

The patient in this scenario is in a third-degree (complete) AV block, which is causing his signs and symptoms. Complete heart block should be treated with immediate transcutaneous cardiac pacing (TCP). Given the patient's clinical presentation, it is clear that he is hemodynamically unstable; obtaining a complete set of vital signs will yield very little, if any, additional information. A 12-lead ECG should be obtained, but not before addressing the most immediate problem of hemodynamic compromise. Atropine should be avoided in patients with high-grade AV heart blocks (eg, second-degree AV block type II and third-degree AV block). Atropine may worsen the patient's condition - especially in cases of third-degree AV block - by increasing sinus node discharge without any effect on the ventricles. Remember, if the rhythm is perfusing, but is slow and wide, begin TCP without delay.

In contrast to the pneuomotaxic center of the medulla, the apneustic center: A) is responsible for setting a person's resting respiratory rate. B) is responsible for terminating inspiration to prevent overexpansion of the lungs. C) influences the respiratory rate by increasing the number of inspirations per minute. D) decreases its influence in times of increased demand by increasing the respiratory rate.

The respiratory center in the medulla is divided into three regions: the respiratory rhythmicity center, the apneustic center, and the pneumotaxic center. The respiratory rhythmicity center sets the resting respiratory rate. The apneustic center influences the respiratory rate by increasing the number of inspirations per minute. Its activity is countered by the pneumotaxic center, which inhibits inspiration. In times of increased demand, the pneumotaxic center decreases its influence, thereby increasing the respiratory rate. As the chest wall expands, mechanical (stretch) receptors in the lungs send a signal to the apneustic center via the vagus nerve to inhibit the inspiratory center, and expiration occurs. This feedback loop, which combines neural and mechanical control, is called the Hering-Breuer reflex. It is a protective mechanism that terminates inspiration, thus preventing overexpansion of the lungs.

A 39-year-old man is unresponsive, pulseless, and apneic after being lost in the woods during the middle of winter. He has a core body temperature of 85° F (29.4° C). When treating this patient, you should avoid: A) defibrillation. B) intubation. C) passive rewarming. D) cardiac medications.

Treatment for patients with cardiac arrest and severe hypothermia (core body temperature less than 86° F [30° C]) includes CPR, intubation (or an alternative airway device), limiting defibrillation to one attempt if the patient is in ventricular fibrillation or pulseless ventricular tachycardia, passive rewarming, and active external rewarming (ie, heating blankets, radiant heat from hot packs). Active internal (core) rewarming is usually performed at the hospital. Cardiac medications should be withheld for two reasons: 1) the patient's metabolic rate is too slow to distribute the drugs, and 2) medications can accumulate to toxic levels in the severely hypothermic patient, which can be detrimental as the patient is rewarmed.

A 47-year-old male took two of his prescribed nitroglycerin tablets prior to calling EMS. When you arrive at the scene, the patient tells you that he has a throbbing headache and is still experiencing chest pain. Your MOST immediate suspicion should be that: A) his chest pain is probably not of a cardiac origin. B) he is experiencing continued myocardial ischemia. C) his nitroglycerin is outdated or has lost its potency. D) permanent myocardial damage has already occurred.

When a patient reports taking nitroglycerin (NTG) for chest pain, you should determine how many tablets or sprays he or she took, and whether or not the NTG relieved his or her pain. Failure of NTG to relieve cardiac-related chest pain can occur for one of two reasons - the pain is of extraordinary severity, such as that associated with acute myocardial infarction, or the NTG has been open too long and has lost its potency. Fresh, potent NTG has certain distinct side effects, including a throbbing headache, a burning sensation under the tongue, and a bitter taste. If the patient did not experience any of these side effects, chances are the drug was outdated or had lost its potency. However, if the patient experienced any of these side effects, but is still experiencing chest pain, you should suspect that he or she is experiencing continued myocardial ischemia and is in the process of having an acute myocardial infarction. A 12-lead ECG and other diagnostic tests (ie, echocardiography) are required to determine if permanent myocardial damage has occurred.

When released into the bloodstream, glucagon: A) directly increases circulating blood glucose levels. B) stimulates the liver to convert glycogen to glucose. C) stimulates the liver to take in glucose and store it as glycogen. D) facilitates the cellular uptake of glucose for energy production.

When the body's blood glucose level falls, such as between meals, glucagon is released into the bloodstream in order to raise the glucose level. Glucagon, a hormone secreted by alpha cells of the islets of Langerhans (the endocrine component of the pancreas), stimulates the liver to convert glycogen to glucose - a process called glycogenolysis - and then releases it into the bloodstream. Through the process of glycogenolysis, glucagon indirectly increases circulating blood glucose levels. Fifty-percent dextrose (D50) is injected directly into the bloodstream; therefore, it directly increases circulating blood glucose levels. Insulin is responsible for the removal of glucose from the blood for storage of glycogen, fats, and protein. When blood glucose levels are elevated, the beta cells of the islets of Langerhans secrete insulin, which is carried by the bloodstream to the cells. The cells then take in more glucose and use it to produce energy. Insulin also stimulates the liver to take in more glucose and store it as glycogen for later use. Insulin is the only hormone that directly lowers blood glucose levels, and is essential for glucose to enter and nourish the cells. Once blood glucose levels have returned to normal, the islets of Langerhans decrease the secretion of insulin.

ECG indicators of Wolff-Parkinson-White (WPW) syndrome include: A) narrow QRS complexes and peaked T waves. B) delta waves, flattened T waves, and bradycardia. C) short PR intervals, delta waves, and QRS widening. D) tall P waves, QT interval prolongation, and tachycardia.

Wolff-Parkinson-White (WPW) syndrome is a condition in which accessory pathways - called the bundle of Kent - bypass the atrioventricular (AV) node, causing the ventricles to depolarize earlier than normal (preexcitation). Because the normal delay at the AV node does not occur, the PR intervals in patients with WPW are usually less than 0.12 seconds (120 ms). When conduction occurs down the AV node and simultaneously along the bundle of Kent in an anterograde fashion, the two waves of depolarization meet (fusion). This manifests on the ECG as a delta wave - slurring or notching at the beginning of the QRS complex - which may cause QRS widening. The bundle of Kent is a potential site for a reentry circuit because it allows continued transmission of an electrical impulse from the atria to the ventricles. Therefore, patients with WPW are prone to reentry tachycardias - most notably, AV reentry supraventricular tachycardia (SVT).

Factors that increase the amount of energy needed for ventilation include all of the following, EXCEPT: A) surfactant deficiency. B) increased airway resistance. C) stimulation of beta-2 receptors. D) decreased pulmonary compliance.

['The amount of energy needed for normal (unassisted) ventilation in an otherwise healthy adult is only about 3% of the total body expenditure. Stimulation of beta-2 receptors would facilitate ventilation by dilating the bronchioles, and would not increase the amount of energy required for ventilation. Anything that impairs ventilation - loss of pulmonary surfactant (eg, emphysema), increased airway resistance (eg, bronchospasm), and decreased pulmonary compliance (eg, pulmonary edema, COPD) - can cause a significant increase in the amount of energy needed for ventilation, perhaps as high as 35%.


Conjuntos de estudio relacionados

Disorders of the skin; Burns & infectious agents

View Set

RSM: Introduction to Business Midterm

View Set

Saunders NClex-PN Fun: Infection Control

View Set

California - Real Estate Practice Ch. 15

View Set

Chapter 42 PrepU: Structure and Function of the Male Genitourinary System

View Set

22.2 Complete the sentences using will ('ll). Choose from the following:

View Set